SlideShare a Scribd company logo
1. A 70-year-old patient has diabetes mellitus and hypertension. He presents
with late-stage chronic kidney disease. Which of the following anti-diabetic
drugs require least dose modification in renal disease?
A. Sitagliptin
B. Vildagliptin
C. Linagliptin
D. Saxagliptin
2. Which of the following drug can cause the side-effect given in the
picture?
A. Bleomycin
B. Mitomycin C
C. Doxorubicin
D. Actinomycin D
3. A patient was on metoprolol therapy for hypertension. He was also
administered verapamil. Which of the following is likely to happen with
above combination therapy?
A. Torsades de pointes
B. Ventricular tachycardia
C. Ventricular fibrillation
D. Bradycardia with AV block
4. All of the following are advantages of enteric coated tablets except
A.It increases the half-life of the drug
B. It protects acid labile drugs from gastric pH
C. It increases the absorption of drugs that are preferentially absorbed distal
to stomach.
D.It protects stomach from irritant drugs
5. A psychotropic drug inhibiting the reuptake of serotonin is likely to cause
which of the following adverse effect?
A. Visual disturbance
B. Constipation
C. Dry mouth
D. Sexual dysfunction
6. Which of the following is a second line anti-tubercular drug?
A.Streptomycin
B. Ethionamide
C. Vancomycin
D.Acyclovir
7. A female with unilateral breast cancer is on tamoxifen therapy. Which of
the following adverse effect is associated with the use of tamoxifen?
A. Carcinoma in opposite breast
B. Endometrial carcinoma
C. Ovarian carcinoma
D. Chronic myeloid leukemia
8. Combined oral contraceptive pills act by all of the following mechanisms
except?
A.Inhibit implantation by bringing change in the uterus
B. Inhibit ovulation
C. Makes cervical mucus thick and hostile
D.Increase gonadotropin secretion
9. Which of the following drug should be avoided in a patient on
rosuvastatin therapy?
A. Clarithromycin
B. Oxycodone
C. Rivaroxaban
D. Adalimumab
10. All of the following are bactericidal drugs against Mycobacterium except
A.Kanamycin
B. Thioacetazone
C. Rifapentine
D.Isoniazid
11. A patient is on long term anticonvulsant therapy. There is progressive
contractility of visual field. The likely anti-epileptic drug responsible for this
adverse effect is
A.Vigabatrin
B. Ethosuximide
C. Phenobarbital
D.Levetiracetam
12. Which of the following drugs act through intracellular receptors?
A.Thyroxin
B. Glucagon
C. Epinephrine
D.Parathyroid hormone
13. Drug of choice for treatment of hairy cell leukemia is?
A.Interferon alpha
B. Fludarabine
C. Pentostatin
D.Cladribine
14. Which of the following drug can be used as an antidote for fibrinolytic
overdose?
A.Ethamsylate
B. Alteplase
C. Andexanet alpha
D.Epsilon amino caproic acid
15. Which of the following action is expected on stimulation of muscarinic receptors?
A. Erection
B. Ejaculation
C. Increased contraction of cardiac muscles
D. Bronchodilation
16. All of the following drugs are indicated in the treatment of NSTEMI
except
A. Clopidogrel
B. Aspirin
C. Streptokinase
D. Prasugrel
17. A 28 year old female with Graves’ disease was taking medication for
hyperthyroidism during pregnancy. She delivered a child with congenital
anomaly ‘aplasia cutis congenita’. Most likely drug implicated for this
teratogenic effect is?
A.Carbimazole
B. Levo-thyroxine
C. Methylthiouracil
D.Liothyronine
18. Which of the following antifungal drugs possess additional anti-
inflammatory and antipruritic activity?
A.Terbinafine
B. Sertaconazole
C. Luliconazole
D.Ketoconazole
19. A 30 year old patient was started on lithium carbonate for bipolar
disorder. The nurse will gave instructions on adverse effects of the therapy.
The patient should report back if the following features are seen:
A.Palpitations, chest pain, raised blood pressure
B. Sedation, constipation, vomiting
C. Tinnitus, severe diarrhea, ataxia
D.Fever, skin rash, bradycardia
20. For plasma level monitoring, lithium estimation is done?
A.Immediately after the dose
B. 8 hours after last dose
C. 12 hours after last dose
D.24 hours after last dose
21. Dicumarol antagonizes the action of vitamin K by
A.Competitive inhibition
B. Non-competitive inhibition
C. Uncompetitive inhibition
D.Suicide inhibition
22. Which of the following is not used in treatment of polycystic ovarian
disease?
A.Letrozole
B. Clomiphene citrate
C. Ulipristal
D.Combined oral contraceptives
23. Most potent opioid is
a. Fentanyl
b. Morphine
c. Pentazocine
d. Pethidine
24. Mechanism of action of allopurinol is
A. Xanthine oxidase inhibition
B. Recombinant uricase
C. Increase uric acid excretion
D. Decrease chemotaxis
25. Which is the following drug causes
postural hypotension commonly?
A. Beta blocker
B. Alpha blocker
C. ACE inhibitor
D. Angiotensin receptor blockers
26. Which of the following drug is a carbonic
anhydrase inhibitor?
A. Acetazolamide
B. Hydrochlorthiazide
C. Furosemide
D. Mannitol
27. Which is most cardiotoxic anticancer
drug?
A. Anthracyclines
B. Cyclophosphamide
C. Imatinib
D. Tamoxifen
28. All of the following drugs are used for
prophylaxis of migraine except
A. Propranolol
B. Imipramine
C. Flutamide
D. Flunarizine
29. A farmer presented with pin point pupil and
increase in secretions. Likely poisoning is
A. Organophosphate poisoning
B. Dhatura poisoning
C. Opioid poisoning
D. Atropine poisoning
30. Which of the following is an oral direct
thrombin inhibitor?
A. Dabigatran
B. Rivaroxaban
C. Warfarin
D. Lepirudin
31. Which of the following drugs follow
zero order kinetics?
A. Phenytoin
B. Aspirin
C. Morphine
D. Propranolol
32. A lady has taken medication of ameiobiasis infection. In a
party she drank alcohol. She has nausea vomiting and
dizziness. Which anti-amoebic drug could have lead to
interaction with alcohol to produce these symptoms?
A. Metronidazole
B. Nitazoxanide
C. Paromomycin
D. Diloxanide
33. Which anti-asthma drug is avoided
with Erythromycin?
A. Salbutamol
B. Theophylline
C. Terbutaline
D. Ipratropium
34. A patient after valve replacement will
require follow up treatment with
A. Warfarin
B. ACE inhibitors
C. Beta blockers
D. Thiazides
35. Antidote for opioid poisoning is
A. Naloxone
B. Pethidine
C. Flumazenil
D. Physostigmine
36. Monoclonal antibody used in cancer is
A. Rituximab
B. Cisplatin
C. 5-Fluorouracil
D. Methotrexate
37. Find therapeutic index of a drug from
the information given below in the graph
A. 1
B. 2
C. 4
D. 8
38. A 34 weeks pregnant lady presented with seizures.
Her blood pressure is 200/110 mm Hg. Which drugs
should be used for the treatment of this patient?
A. Oral Labetalol
B. IV labetalol plus magnesium sulphate
C. IV Lorazepam
D. IV Enalapril
39. Hydrochlorothiazide works by
inhibiting
A. Na+ Cl- pump in DCT
B. Na+ Cl- pump in PCT
C. Na+ K+ 2 Cl- pump in descending limb of loop of Henle
D. Na+ K+ 2 Cl- pump in ascending limb of loop of Henle
40. Drug of choice for hyperthyroidism in third trimester of
pregnancy is
A. Carbimazole
B. Propylthyouracil
C. Sodium iodide
D. Radioactive iodine
41. Mechanism of action of exenatide in diabetes mellitus is
A. It is analogue of GLP released from gut and increase glucose
dependent insulin secretion
B. It is DPP-4 inhibitor and result in decreased breadkdown of GLP
C. It inhibits SGLT-2 and cause glucosuria
D. It is amylin analogue and decrease glucagon
42. Least teratogenic antiepileptic drug in pregnancy is
A. Valproate
B. Phenytoin
C. Carbamazepine
D. Levetiracetam
43. A person presented to emergency in comatose state after consuming some
unknown poison. Excessive sweating is apparent and on examination there is
bradycardia and BP is 80/60 mmHg. Which of the following is antidote for
treatment of this patient?
a. Neostigmine
b. Atropine
c. Physostigmine
d. N-acetylcysteine
44. Drug used for treatment of pregnancy induced hypertension is
a. Alpha methyl dopa
b. Lisinopril
c. Hydralazine
d. Telmisartan
45. A 56-year-old patient who is a known case of hypertension was being treated
with antihypertensive drugs. He is having chronic cough since 2 weeks. He was
started on antimicrobials but cough did not resolve. Which of the following
antihypertensive drug may induce chronic cough in this patient?
a. Lisinopril
b. Minoxidil
c. Propranolol
d. Amlodipine
46. A patient presented with unilateral hemiparesis and presented to hospital
after 8 hours. On examination and investigations, the blood pressure is 220/110
mmHg and HbA1c is 8%. CT scan of brain shows no hemorrhage. Which is the
appropriate management of this patient?
a. Conservative management
b. tPA
c. IV labetalol
d. Sublingual nifedipine
47. A professor in medical college is teaching students about the adverse effects
of a drug. Which of the following drug can cause gynaecomastia in men and
menstrual irregularities in women?
a. Spironolactone
b. Propranolol
c. Misoprostol
d. Oxytocin`
48. A 25 year old male presented to emergency with head trauma due to road
traffic accident. In hospital, the patient developed seizures. Emergency CT scan
revealed widespread cerebral edema. Which of the following is diuretic of choice
for cerebral edema in this patient?
a. Mannitol
b. Spironolactone
c. Furosemide
d. Hydrochlorthiazide
49. A 28 year old male went to Ladakh. He developed shortness of breath. Which
of the following drug will you use for the treatment of mountain sickness at high
altitude?
a. Acetazolamide
b. Furosemide
c. Epleronone
d. Salbutamol
50. A patient was operated for prostate carcinoma and during surgery local
metastasis was noted. Which of the following drug this patient should receive?
a. Leuprolide
b. Desmopressin
c. Octreotide
d. Tamsulosin
51. A 30-year-old female presented with weight gain and cold intolerance. On
investigations Value of T3 is 70 ng/dl (reference value 100-200 ng/dl),T4 is 3
mcg/dl (reference value 5-12 mcg/dl) and TSH is 20 mIU/L (reference value 0.5-5
mIU/L). Which of the following statement is correct regarding the management of
this patient?
a. Measure T3, T4 and TSH after one month
b. Start L-thyroxine (T4) treatment 100 mcg once daily
c. Start L-thyroxine (T4) treatment 25 mcg daily and gradually increase
d. Start daily L-thyroxine (T4) 100 mcg with Liothyronine (T3) 5 mcg
52. A 50-year-old patient of type 2 diabetes mellitus was controlled on oral
hypoglycemic drugs. This patient presented to hospital with profuse sweating
and dizziness. There was presence of hypoglycemia. Which among the following
drugs can result in hypoglycemia in this patient?
a. Metformin
b. Voglibose
c. Vildagliptin
d. Glipizide
53. Which of the following statements about the anti-psychotic
drug, Aripiprazole is most appropriate?
a. It is a D2 and 5HT 1A partial agonist with less extrapyramidal symptoms and less weight
gain as compared to other anti-psychotic drugs
b. It is a D2 and 5HT 1A partial agonist with more extrapyramidal symptoms and more
weight gain as compared to other anti-psychotic drugs
c. It is a D2 antagonist and 5HT2 agonist with less extrapyramidal symptoms and less weight
gain as compared to other anti-psychotic drugs
d. It is a D2 antagonist and 5HT2 agonist with less extrapyramidal symptoms and less weight
gain as compared to other anti-psychotic drugs
54. A young boy was brought to hospital with complaints of few episodes of fast
blinking and staring at something for around 20 seconds. ECG shows spike and
wave pattern. Which of the following drug should be used for treatment?
a. Phenytoin
b. Sodium valproate
c. Diazepam
d. Carbamazepine
55. Which of the following options contains all intravenous hypnotic agents?
a. Propofol, sodium thiopentone, Etomidate
b. Halothane, sevoflurane, desflurane
c. Suggamadex, Sufentanil, tramadol
d. Vecuronium, Rocuronium, Atracurium
56. What is the mechanism of action of tPA?
a. Fibrinolysis
b. Inhibition of factor V and VII
c. Inhibition of factor III and V
d. Activation of anti-thrombin III
57. A patient with chronic obstructive pulmonary disease is on salmeterol
therapy but there is no improvement. Which of the following drug should be
added to the regimen?
a. Inhaled Corticosteroids
b. Tiotropium
c. Oxygen therapy
d. Promethazine
58. A patient is taking platinum-based chemotherapy for treatment of ovarian
cancer. Which of the following drug should be used to prevent the nausea and
vomiting in this patient?
a. Domperidone
b. Promethazine
c. Palonosetron
d. Metoclopramide
59. A patient has developed tuberculosis in which mycobacterium is resistant to
rifampicin, isoniazid, kanamycin and levofloxacin. This tuberculosis is referred to
as
a. MDR TB
b. Rifampicin resistant TB
c. Extensively resistant TB
d. Extremely resistant TB
60. A female patient presented with greenish vaginal discharge. Clue cells were
present and Whiff test was positive in vaginal discharge. What is the drug of
choice for this condition?
a. Doxycycline
b. Metronidazole
c. Ceftriaxone
d. Fluconazole
61. Which of the following drug will you omit from the prescription of a patient
with XDR tuberculosis in pregnancy?
a. Kanamycin
b. Ethambutol
c. Moxifloxacin
d. Amoxycillin-clavulanic acid
62. A female patient was treated with clindamycin and developed diarrhea most
probably pseudomembranous colitis. Which of the following is the likely
organism responsible for this?
a. Clostridium perfringens
b. Clostridium difficile
c. Clostridium tetani
d. Clostridium septicum
63. Mechanism of action of doxycycline is
a. DNA gyrase inhibitor
b. Cell wall synthesis inhibitor
c. Protein synthesis inhibitor
d. DNA dependent RNA polymerase inhibitor
64. A 30-year-old man travelled to Assam. After coming back he developed fever,
chills and rigors and altered sensorium. The peripheral smear demonstrated the
presence of P. falciparum. What is the treatment of choice for this patient?
a. Chloroquine, Sulfadoxine pyrimethamine
b. Intravenous Artesunate
c. Quinine
d. Primaquine
65. A 50-year-old female presented to oncologist with breast lump. On
examination and investigations, breast cancer was diagnosed. Echocardiography
of the patient shows ejection fraction of 25 %. Which of the following drug will
you avoid in this patient?
a. Epirubicin
b. Irinotecan
c. Cyclophosphamide
d. Methotrexate
66. Which of the following is an adverse effect of orlistat?
a. Suicidal tendancy
b. Steatorrhea
c. Weight gain
d. Hyperglycemia
67. What does LD50 signifies
a. It is the dose at which 50% of animals will die
b. It is the dose at which 50 animals will die
c. It is the dose at which 50% patients will get the desired
response
d. It is drug that will be effective at 50% dose
68. All of the following are Side effects of Ritodrine except?
a. Hypoglycemia
b. Tachycardia
c. Hypotension
d. Pulmonary edema
69. A patient with history of snake bite presented in
emergency with symptoms of fatigue and irritability. Ten
vials of antivenom were administered intravenously but the
patient is still having difficulty in swallowing, ptosis and
respiratory distress. What should be the next line of
management?
a. Atropine and neostigmine
b. Nebulizer with salbutamol
c. IV hydrocortisone
d. Antihistamines
70. A known patient of bronchial asthma was started on
treatment with an anti-glaucoma drug. During the course of
treatment, asthma worsened. Which drug is likely
responsible for these symptoms?
a. Timolol
b. Brimonidine
c. Latanoprost
d. Pilocarpine
71. Which of the following is the fastest urate lowering
therapy in tumor lysis syndrome to prevent renal damage?
a. Allopurinol
b. Febuxostat
c. Rasburicase
d. Furosemide
72. A patient presented with unstable angina and was
started on 325 mg aspirin. What is correct about mechanism
of action of this drug at this dose?
a. It acts by inhibiting aggregation of platelets
b. It acts by Inhibiting COX-1 in both endothelium and
platelets
c. It acts as a fibrinolytic agent
d. It acts as an anti-coagulant agent
73. A known hypertensive patient came to emergency
department with complaints of palpitations and shortness of
breath. Her BP was 220/140. Among the option given below,
which drug should not be used in treatment of this patient?
a. I.V Nitro prusside
b. I.V Esmolol
c. I.V. Nicardipine
d. I.V.Mannitol
74. A patient with chronic stable angina presents with pedal
edema, breathlessness and other symptoms of heart failure.
Which of the following drug can increase longevity in this
patient?
a. Nifedipine
b. Digoxin
c. Lisinopril
d. Torsemide
75. A female washing clothes in cold water exhibited color
change of her hands from pallor to red. Which of the
following drug is best suited for this patient?
a. ACE inhibitor
b. Calcium channel blockers
c. Thiazides
d. Alpha blockers
76. What is the Drug of choice for Hyperthyroidism in first
trimester of Pregnancy?
a. Phosphorus iodine
b. Methimazole
c. Propylthiouracil
d. Carbimazole
77. To minimize the risk of HPA axis suppression, What is the
correct method of administration of prednisolone?
a. Give at night just before bedtime
b. Divide in three doses and give small doses 8 hourly
c. Give on alternate day
d. Replace with betamethasone
78. Which of the following drug used for pain relief in
arthritis acts by both opioid and non opioid mechanisms?
a. Paracetamol
b. Tramadol
c. Ketorolac
d. Diclofenac
79. A patient with history of drug abuse presented to
emergency in comatose state. Vitals recorded were;
Respiratory rate 8/min, BP: 90/60 mmHg, Pulse 52 bpm. On
examination, there was pin point pupil. What is the
probable poisoning agent in this patient?
a. Cocaine
b. Morphine
c. Alcohol
d. Tricyclic antidepressant
80. A female visits the OPD to get rid of tobacco addiction
Which of the following drug will you prescribe?
a. Vareniciline
b. Naloxone
c. Disulfiram
d. Flumazenil
81. A patient was started on lamotrigine 50 mg daily for
treatment of epilepsy. Gradually, the dose of the drug is
increased every 1-2 weeks to reach the final dose of 200 mg
per day. This step up titration of lamotrigine dose is done in
order to prevent or minimize the development of
a. Sedation
b. Sudden death
c. Weight gain
d. Skin rash
82. A 25-year-old female presented with mania. History
revealed that her father is taking medications for bipolar
disorder. Which of the following drug will you prescribe for
this patient?
a. Sodium valproate
b. Lithium
c. Carbamazepine
d. Lamotrigine
83. A patient was administered a local anaesthetic agent and
developed tachycardia, hypotension, arrhythmias and
cardiovascular collapse. Which of the following should be
used for treatment of this patient?
a. 5% dextrose
b. 20% dextrose
c. 5% intralipid
d. 20% intralipid
84. A patient presented to hospital with chest pain. ECG of
the patient confirms it to be non ST elevation myocardial
infarction. Which of the following drug useful in this patient
acts by inhibiting platelet aggregation?
a. Clopidogrel
b. Streptokinase
c. Low molecular weight heparin
d. Warfarin
85. Which of the following anticoagulant is contraindicated
in first trimester of pregnancy?
a. Low molecular weight heparin
b. Warfarin
c. Unfractionated Heparin
d. Enoxaparin
86. A 25-year-old male has frequent visits to rural areas and
need to eat outside food. During one such visit, he
developed loose stools. Which of the following drug should
be used to manage this case?
a. Metronidazole
b. Diphenoxylate
c. Loperamide
d. Octreotide
87. Drug of choice for Legionnaire’s disease is:
a. Azithromycin
b. Tigecycline
c. Streptomycin
d. Amoxycillin
88. A patient developed COVID-19 pneumonia and was
treated. Few months later, patient presented with nasal
congestion and swelling of face. There were black lesions on
the nose bridge. A diagnosis of mucormycosis was made.
What is the drug of choice for this condition?
a. Fluconazole
b. Dexamethasone
c. Amphotericin B
d. Azithromycin
89. A female patient was taking warfarin for prosthetic
heart valves. She became pregnant. What advise will you
give regarding use of this drug in early pregnancy?
A. Continue warfarin without any concern
B. Continue warfarin but add vitamin K to the therapy
C. Change warfarin to heparin
D. Terminate the pregnancy
90. Which of the following drugs used in emergency
management of asthma does not act by causing
bronchodilaton?
A. Salbutamol
B. Ipratropium
C. Deriphylline
D. Hydrocortisone
91. Which of the following is the action of
dopamine at low doses?
A. It increases renal blood flow
B. It causes vasoconstriction
C. It has positive chronotropic action
D. It causes hypotension
92. A female patient presented with pain and redness in
great toe. Serum uric acid level is 9.6 mg/dL. Apart from
prescribing analgesics for relieving pain, the physician
prescribed a drug which can decrease the formation of
uric acid. Which of the following enzyme is likely to be
inhibited by this drug?
A. Xanthine oxidase
B. Thymidylate synthase
C. Phosphpribosyl transferase
D. DHFR
93. Vitamin that increases the absorption
of iron from stomach is?
A. Vitamin A
B. Vitamin B12
C. Vitamin C
D.Vitamin D
94. Longest acting phosphodiesterase inhibtor
among the following drugs is?
A. Sildenafil
B. Vardenafil
C. Tadalafil
D. Phentolamine
95. Which of the following statements about
these drugs is true?
A. Drug A is most potent
B. Drug A and B have equal
efficacy but B is more
potent than drug A
C. Drug C is least potent
D. Drug B has more potency
but less efficacy than drug
A
96. Which of the following anti-depressant drug
acts by inhibiting the reuptake of both serotonin
and nor-adrenaline?
A. Fluoxetine
B. Venlafaxine
C. Phenelzine
D. Nortriptyline
97. A 4-year-old boy drank full bottle of
iron syrup. What is the antidote for acute
iron poisoning?
A. BAL
B. EDTA
C. Desferrioxamine
D.Penicillamine
98. Which of the following antimicrobials is
contraindicated in a patient with seizure
disorder?
A. Ampicillin
B. Ofloxacin
C. Doxycycline
D.Cefixime
99. Which of the following drug is preferred for
closure of ductus arteriosus in a preterm baby
with PDA?
A. Indomethacin
B. Ibuprofen
C. Mefenamic acid
D.Paracetamol
100. Drug of choice for treatment of
digoxin induced ventricular arrhythmias is?
A. Atropine
B. Lidocaine
C. Amiodarone
D.Procainamide
101. A patient was given halothane for general
anaesthesia. He developed rigidity and
hyperthermia. These symptoms are most likely
due to which ion?
A. Na+
B. K+
C. Ca2+
D.Cl–
102. A lady was stung by many bees. She
developed breathing difficulty and
hypotension. Drug used for treatment of this
condition is?
A. Adrenaline
B. Nor-adrenaline
C. Dopamine
D.Isoprenaline
103. A patient presented to hospital with road traffic
accident. The patient has raised ICP without
intracranial hematoma. Which of the following is the
preferred drug for the management of this patient?
A. Mannitol
B. Prednisolone
C. Glycerol
D.Furosemide
104. A patient of COVID-19 requires oxygen therapy.
Which of the following cylinders should be used to
provide oxygen to this patient?
A B C D
105. A drug addict patient presented with agitation
and ulceration on skin. The patient had severe itching
and scratch marks on the body. On examination there
is presence of tachycardia, mydriasis and
hypertension. Likely substance responsible for these
symptoms is?
A. Cocaine
B. Cannabis
C. LSD
D.Heroin
106. A patient on treatment of myasthenia gravis
presents with increasing muscle weakness. Which of
the following drug is used to differentiate
myasthenia gravis from cholinergic crisis?
A. Edrophonium
B. Pilocarpine
C. Atropine
D.Pyridostigmine
107. A patient had a history of previous child with
down syndrome, currently on lithium therapy
present for antenatal checkup. Which of the
following condition is the fetus at the greatest risk
of?
A. Ebstein’s anomaly
B. Down’s syndrome
C. Tetralogy of Fallot
D.Neural tube defect
108. Hormones and endogenous substances that
are involved in regulation of calcium balance in
the body are?
A. Vitamin D, PTH and insulin
B. Vitamin D, PTH and calcitonin
C. Vitamin D, calcitonin and TSH
D. Vitamin D, PTH and TSH
109. Which of the following drugs can be used
for the treatment of undescended testes?
A. Testosterone
B. GnRH
C. Cold water compress
D.Anti-MIH therapy
110. After eating an unknown plant, A child
presented to emergency in comatose state. The
pupils were dilated and skin was dry. Body
temperature is raised and heart rate is high. These
signs and symptoms can occur with the intake of
which of the following substance?
A. Dhatura
B. Poppy seeds
C. Mushroom
D.Cannabis
111. A farmer presented to emergency with garlic
odour, pin point pupil and excessive secretions.
Which of the following is drug of choice for
treatment of this poisoning?
A. Atropine
B. Flumazenil
C. N-acetylcysteine
D.Oxime
112. An elderly male developed the symptoms
of dementia and was diagnosed as Alzhiemer’s
disease. Which neurotransmitter level is
reduced in this disease?
A. Dopamine
B. Acetylcholine
C. Adrenaline
D.Serotonin
113. An elderly patient presented with tremors,
rigidity and bradykinesia. The most likely
neurotransmitter affected in the brain of this patient
is?
A. Dopamine
B. Acetylcholine
C. Adrenaline
D.Serotonin
114. Which of the following combination of
seizure and its first line drug is not correct?
A. Generalised seizure - Valproate
B. Myoclonic - Topiramate
C. Focal - Levetiracetam
D.Absence - Ethosuximide
115. Aspirin may increase serum uric acid. What is the mechanism for this
action?
A. It inhibits reabsorption of uric acid from renal tubules
B. It results in increased production of uric acid
C. It interferes with filtration of uric acid through glomerulus
D. It causes breakdown of proteins leading to excessive uric acid
production
116. Which of the following drug is given to mother delivering premature
baby for fetal lung maturation?
A. Aspirin
B. Dexamethasone
C. Magnesium sulfate
D. Depot medroxy progesterone acetate
117. A patient presented with hospital acquired pneumonia. On testing he
was found to be allergic to penicillin G. Which of the following drug is
likely to be safe in this patient?
A. Ceftriaxone
B. Ampicillin
C. Aztreonam
D. Imipenem
118. A patient was taking theophylline for bronchial asthma. After starting
another drug, symptoms of theophylline toxicity started appearing.
Which is the likely drug started?
A. Erythromycin
B. Phenytoin
C. Griseofulvin
D. Rifampicin
119. Which intravenous anaesthetic is used for day care surgery?
A. Ketamine
B. Propofol
C. Thiopentone
D. Etomdiate
120. Which of the following drug is associated with post operative delirium
and hallucinations?
A. Ketamine
B. B. Thiopentone
C. C. Fentanyl
D. D. Halothane
121. A 65 years old patient presented with dribbling of urine with urgency.
He was started on prazosin therapy but the patient developed
postural hypotension. Which of the following is the better alternative
drug for this patient?
A. Terazosin
B. Tamsulosin
C. Timolol
D. Phenoxybenzamine
122. Which of the following is a selective estrogen receptor modulator?
A. Tamoxifen
B. Anastrozole
C. Mifepristone
D. Ethinyl estradiol
123. Which among the given options is a monoclonal antibody used for
treatment of cancer?
A. Cisplatin
B. Rituximab
C. Vincristine
D. Cyclophosphamide
124. A new drug is introduced in the market after which phase of clinical
trials
A. Phase 1
B. Phase 2
C. Phase 3
D. Phase 4
125. Identify the missing substance X marked in the box in the given
diagram.
A. G protein
B. Protein C
C. Cyclic AMP
D. Calcium
126. In the given diagram, identify drug Z marked by the arrow
A. Clopidogrel
B. Heparin
C. Dicumarol
D. Alteplase
127. Excessive phytates intake in the diet may lead to deficiency of
A. Proteins
B. Iron
C. Vitamin B12
D. Vitamin D
128. Beta blockers are avoided in all of the following conditions except
A. Glaucoma
B. Peripheral vascular disease
C. Diabetes mellitus
D. COPD
129. A 22 year old female presents 8 hours after sexual assault. It is the
13th day of her menstrual cycle. Which emergency contraceptive
should be prescribed to her?
A. Levonorgestrel 1.5 g single tablet
B. Injection DMPA
C. OCP from day 1 of next cycle
D. Misoprostol
130. A patient presented with the following features after chronic intake
of a drug for several years. Which is the likely drug person has
consumed?
A. Paracetamol
B. Prednisolone
C. Phenytoin
D. Metformin
131. Which of the following is wrongly matched?
a.Dabigatran: Idarucizumab
b.Rivaroxaban: Andexanet alpha
c.Fondaparinux: Ciraparantag
d.Apixaban: Andexanet alpha
132. A patient had symptoms of redness and photophobia. On
examination, cells were present in anterior chamber with keratic
precipitates. Intraocular pressure was measured to be 38 mm Hg.
Which of the following anti-glaucoma drug should be avoided?
a.PG analogues
b.Beta blockers
c.Mannitol
d.Carbonic anhydrase inhibitors
133. A hypertensive patient has grade 4 renal failure and
GFR less than 30 mL/min. The physician wants to
prescribe a thiazide diuretic. Which is the best drug for
this patient?
a.Hydrochlorthiazide
b.Chlorthalidone
c.Metolazone
d.Indapamide
134. A study was conducted to see the effect of different drugs on
isolated mammalian intestinal tissue in Dale’s organ bath. The following
graph is obtained. Which of the following is the likely drug?
a.Acetylcholine
b.Barium chloride
c.Adrenaline
d.KCl
135. Nivolumab is a monoclonal antibody used for the
treatment of
a.Hodgkin’s lymphoma
b.Medulloblastoma
c.Retinoblastoma
d.Pleuropulmonary blastoma
136. A patient was taking chemotherapy cyclophosphamide,
methotrexate and 5-Fluorouracil for treatment of breast cancer.
She developed fever and on investigations, She had anemia and
neutropenia. Which of the following antimicrobial is not indicated
in this patient for treatment of this infection?
a.Piperacillin-tazobactam
b.Cefepime
c.Linezolide
d.Meropenem
137. A patient on retigabine therapy for a month for focal
seizures. Phenytoin was added to therapy. What is the next
step?
a.Change retigabine to carbamazepine
b.Decrease dose
c.Increase dose
d.Stop retigabine
138. Which of the following drugs can be used for the
treatment of severe covid-19 pneumonia in children?
a.Steroids
b.Remdesivir
c.Ivermectin
d.All of these
139. We administer a drug following first order kinetics. If the
administered dose is doubled?
a.Plasma concentration and elimination half life remains same
b.Plasma concentration becomes double and elimination half life
remains same
c.Elimination half life becomes double and plasma concentration
remains same
d.Elimination half life and plasma concentration both becomes
double
140. Identify the mechanism of action of vancomycin from the
given figure:
141. For treatment of which of the following conditions,
a combination of antimicrobials is usually not required?
a.Malaria
b.Gonorrhea
c.Intra-abdominal infection
d.Tuberculosis
142. Topiramate is used for the treatment of
a.Lennox Gastaut syndrome
b.Attention deficit hyperkinetic disorder
c.Treatment of migraine
d.Prophylaxis of heat stroke
143. Which of the following is FDA-approved drug for long
term treatment of obesity?
a.Sibutramine
b.Liraglutide
c.Metformin
d.Fenfluramine
144. A newly diagnosed tuberculosis patients was found to be
HIV positive. Which of the following statement is true about
treatment of this patient?
a. Start ATT and ART together
b. ART should be started only if CD4 count is less than 50/mcL
c. ATT should be started first followed by ART 2 weeks later
d. ART should be started first followed by ATT 2 weeks later
145. A patient with stroke was started on clopidogrel. He
developed another episode of stroke 6 months later. Which
of the following is the likely cause?
a.Rapid metabolism by CYP 1A2
b.Poor metabolism by CYP 2C19
c.Poor metabolism by CYP 2D6
d.Poor metabolism by CYP 2E1
146. Which of the following is true about nicotine replacement
therapy?
a. Vareniciline is a form of NRT that comes with black box
warning of cardiovascular adverse effects
b. There should be a gap of minimum 15 minutes between
nicotine and coffee or acidic food
c. NRT is given by GI route
d. Nicotine gum is better because it attains 10-15 percent higher
plasma concentration than lozenges
147. A patient was given ipratropium and he developed
paradoxical bronchoconstriction. Which is not a likely mechanism
of this bronchoconstriction?
a.Benzalkonium chloride
b.Presynaptic stimulation of M2 autoreceptors?
c.Use of hypertonic saline in nebuliser
d.EDTA
148. Mechanism of action of local anaesthetics is
a.Alters the resting membrane potential of the cell
b.Decrease transport of K across cell
c.Decrease transport of Na via voltage gated channels
d.Binds to receptor complex and prevents activation of gates
149. Which of the following factor in PC vs time graph is a measure of extent of
drug absorption of a drug?
a. Area under the curve
b. Half life
c. Cmax
d. Tmax
150. Manufacturer of a drug company labels the drug contains 500
mg paracetamol. On quantitative analysis by the authorities, it was
found to contain only 200 mg of drug. According to drugs and
cosmetics act 1940, this type of drug is known as?
a. Spurious drug
b. Adulterant drug
c. Unethical drug
d. Misbranded drug
151. Which of the following statement is correct regarding the given graph?
a. Drug A represents agonist and Drug B represents inverse agonist
b. Drug C represents agonist and Drug D is inverse agonist
c. Drug A is agonist and Drug D is inverse agonist
d. Drug B is partial agonist and Drug C is inverse agonist
152. Type of stimulatory G protein in PIP2-Phospholipase activation pathway is
a. Gs
b. Gi
c. Gq
d. Go
153. Identify the type of inhibition from the given graph
a. Competitive
b. Non-competitive
c. Un-competitive
d. Allosteric
154. A 20-year-old boy was brought to emergency room after consuming
some unknown substance. The patient was sweating profusely, saliva was
drooling from mouth and tears were present. The boy had involuntary
urination and diarrhea. On examination, the heart rate was 58 beats per
minute and blood pressure was 80/60 mm Hg. Which of the following is the
drug of choice for treatment of this patient?
a. Atropine
b. N-acetylcysteine
c. Naloxone
d. Physostigmine
155. A patient with pre-existing liver disease consumed a drug and
developed worsening of liver dysfunction. The metabolite of the drug
responsible is N-acetyl-para-amino-benzo-quinone-imine. The likely
implicated drug in this case is
a. Paracetamol
b. Valproate
c. Amiodarone
d. Lorazepam
156. Identify the correct match regarding the drug and its
adverse drug reaction?
1. Hydralazine: Heart failure
2 Verapamil: Constipation
3. Aliskiren: Hypokalemia
4. Atenolol: Hemolytic anemia
157. Which of the following anti-diabetic drugs is associated with increased risk
of fractures in a female with osteoporosis?
a. Canagliflozin
b. Rosiglitazone
c. Voglibose
d. Rapaglinide
158. Half life of letrozole is
a. 45 hours
b. 72 hours
c. 96 hours
d. 120 hours
159. Which of the following statement/statements regarding use of 5-alpha
reductase inhibitors in BPH are correct? (multiple correct)
a. Decrease in serum PSA
b. Increase in serum PSA
c. Decrease in cellular testosterone
d. Decrease in cellular DHT
160. A 42 year old chronic alcoholic presents to emergency department with
altered sensorium and seizures. He has not consumed alcohol for last two days.
On investigations, his serum ALT is 150, AST is 180 and GGT is 563 units per litre.
Best drug for first line management of this patient is?
a. Lorazepam
b. Diazepam
c. Clonazepam
d. Alprazolam
161. ACTH is the treatment of
a. Juvenile myoclonic epilepsy
b. West syndrome
c. Dravet syndrome
d. Lennox Gastaut syndrome
162. A patient of psychosis was being treated with risperidone. He presented to
emergency with upward fixed gaze. What will be the treatment of this patient?
a. Wait and assurance
b. Intramuscular Promethazine
c. Injection Diazepam
d. Injection Lorazepam
163. A 30 year old female presented to psychiatry OPD with symptoms of
hypomania. She has a past history of mania and wants to conceive. Which of the
following drug is most teratogenic?
a. Valproate
b. Lithium
c. Carbamazepine
d. Olanzapine
164. Which of the following options correctly represent the increasing order of
potency of inhalational anaesthetic agents?
a. N2O < Isoflurane < Halothane < Methoxyflurane
b. Methoxyflurane < Halothane < Isoflurane < N2O
c. Halothane < Isoflurane < Methoxyflurane < N2O
d. Isoflurane < N2O < Halothane < Methoxyflurane
165. From the given diagram, identify the mechanism of action of isoniazid?
a. Drug A
b. Drug B
c. Drug C
d. Drug D
166. The given diagram shows the steps in formation of bacterial cell wall.
Identify the site of action of beta lactam antimicrobials.
a. A
b. B
c. C
d. D
167. Pregnant female with history of lepromatous leprosy presents to OPD with
type 2 lepra reaction. What should be the management?
a. Thalidomide
b. Stop MDT and start steroids
c. Add corticosteroids
d. Antibiotics
168. Active metabolite of cyclophosphamide is
a. 4-ketoacyl cyclophosphamide
b. 4-hydroxy cyclophosphamide
c. N-Acetyl cyclophosphamide
d. N-Methylcyclophosphamide
169. Immune checkpoint inhibitor approved for treatment of advanced
endometrial carcinoma is?
a. Pembrolizumab
b. Nivolumab
c. Ipilimumab
d. Trastuzumab
170. Which of the following is not an immune check point inhibitor?
a. Cetuximab
b. Pembrolizumab
c. Atezolizumab
d. Nivolumab
171. Which of the following drug is not indicated in treatment of Sickle cell
anemia?
a. Bebtelovimab
b. L-glutamine
c. Hydroxyurea
d. Voxeletor
172. Elimination rate constant of a drug is 0.05/hr. What is
its half life?
A. 6.5 hr
B. 20 hr
C. 13.9 hr
D. 8 hr
173. Anionic and slightly acidic drugs usually bind to
A. Albumin
B. Alpha acid glycoprotein
C. Ceruloplasmin
D. Globulin
174. The following table gives the data of AUC of drug A
alone and AUC of drug A when combined with drug B. p
value is <0.01. Which of the following statement regarding
these drugs is most correct?
A. Drug B decreases the first pass metabolism of drug A
B. Drug B increases the systemic metabolism of drug A
C. Drug B decreases the intestinal absorption of drug A
D. Drug B increases the renal clearance of drug A
Drug Area Under the Curve (AUC)
A alone 550 ± 150
A plus B 850 ± 150
175. Category A, B, C, D, X division of drugs is based on
A. Safety in pregnancy
B. Dose adjustment in renal failure
C. Therapeutic index and safety
D. Over the counter use of drug
176. Which dose of dopamine act preferably on
beta-1 receptors?
A. Less than 2 mcg
B. 2-10 mcg
C. 10-20 mcg
D. More than 20 mcg
177. A patient presents with grade 2 pulmonary artery
hypertension. Vaso-reactive stimulation test is negative.
Which is the most preferred initial management of this
patient?
A. Epoprostenol
B. Amlodipine
C. Alprostadil
D. Ambrisentan
178. Resistant hypertension is defined as inability to attain
goal blood pressure inspite of the concurrent use of 3
antihypertensive agents of different classes prescribed at
optimal doses including
A. Alpha blockers
B. Diuretics
C. Reserpine
D. Alpha Methyl Dopa
179. A 65-year-old man comes to OPD with history of fall. He is
hypertensive with history of atrial fibrillation and is presently on
captopril, atenolol, aspirin and amiodarone. He presents with the
following finding as shown in the image. What is the most
probable diagnosis?
A. Silver toxicity
B. Lupus pernio
C. Amiodarone induced skin lesion
D. Captopril toxicity
180. A patient of chronic liver cirrhosis presents with ascites.
Which of the following is the best diuretic to be used in this
patient?
A. Eplerenone
B. Chlorthiazide
C. Furosemide
D. Triamterene
181. What is the prenatal dose of dexamethasone given for
lung maturation in premature infants ?
A. 6 mg 2 doses 12 hours apart
B. 12 mg 2 doses 24 hours apart
C. 6 mg 4 doses 12 hours apart
D. 6 mg 4 doses 24 hours apart
182. A 60 year old female presents to hospital for routine
check-up. She attained menopause at 52 years of age and
has a past history of Colle’s fracture. DEXA scan was done
and T-score is -2.5. What should be the treatment given to
this patient?
A. Vitamin D and Calcium supplementation
B. Alendronate
C. Repeat DEXA Scan
D. Hormone replacement therapy
183. Dose of carbetocin for post partum hemorrhage is
A. 50 mcg iv over 2 min
B. 100 mcg iv over 1 min
C. 150 mcg im over 2 min
D. 200 mcg iv over 1 min
184. Which of the following drug is the only medication
useful in preventing disability progression in a patient with
primary progressive multiple sclerosis?
A. Natalizumab
B. Ocrelizumab
C. Siponimod
D. Rituximab
185. Most effective cessation agent when used as
monotherapy for smoking cessation is?
A. Varenicline
B. Nicotine gum
C. Sustained release Bupropion
D. Nicotine Patch
186. Which of the following is a novel antidepressant drug?
A. Vilazodone
B. Asenapine
C. Flibanserin
D. Lurasidone
187. Which of the following antidepressant drug should be
avoided in a patient of angle closure glaucoma?
A. Sertraline
B. Amitriptyline
C. Mirtazapine
D. Venlafaxine
188. Which of the following anti-epileptic drug is used
in treatment of neuropathic pain ?
A. Gabapentin
B. Lamotrigine
C. Carbamazepine
D. Pregabalin
a) A,C,D
b) A,B,C
c) A,B
d) A,C
189. Which of the following drugs improves bioavailability
and prolongs duration of action of Saquinavir?
A. Ritonavir
B. Cimetidine
C. Vitamin C
D. Remdesivir
190. A patient on phenytoin for seizure disorder was
prescribed sucralfate 4 times a day for peptic ulcer.
What should be the minimum duration between
consumption of two drugs?
A. 30 min
B. 60 min
C. 90 min
D. 120 min
191. Mechanism of action of Remdesivir is inhibition of?
A. RNA dependent RNA polymerase
B. DNA dependent RNA polymerase
C. Viral protease enzyme
D. Cell wall synthesis
192. Which of the following cephalosporins can increase the
effect of warfarin resulting in raised INR and increased risk
of bleeding?
A. Cefoperazone
B. Cefixime
C. Ceftibiprole
D. Ceftazidime
193. A female patient presented with greenish vaginal
discharge and pruritus. On colposcopy, strawberry cervix is
noted. What is the drug of choice for this condition?
A. Ceftriaxone
B. Metronidazole
C. Acyclovir
D. Fluconazole
194. What is mechanism of action of Cyclosporine?
A. Calcineurin inhibitor
B. mTOR inhibitor
C. IL-2 receptor antagonist
D. TNF-α inhibitor
195. A patient was given intravenous botulinum toxin and
the patient died. Mechanism of botulinum toxin overdose
is
A. Inhibits release of acetylcholine
B. Inhibit reuptake of nor-epinephrine
C. Blockade of post synaptic nicotinic cholinergic receptors
D. Inhibit entry of acetylcholine in vesicle
196. Drug of choice in paracetamol overdose is
a. N-acetylcysteine
b. Dopamine
c. Hydralazine
d. Furosemide
197. A young female presented with left sided severe
throbbing headache associated with nausea, vomiting,
photophobia and phonophobia. Which of the following
drug can provide immediate relief to this patient?
a. Propranolol
b. Sumatriptan
c. Topiramate
d. Flunarizine
198. A patient on digoxin therapy accidently consumed 8
tablets of digoxin 0.25 mg. Two hours later, he presented
to emergency with heart rate of 54 bpm and ECG
evidence of third-degree AV block. What is the immediate
management of this patient?
a. Digoxin immune Fab
b. Lignocaine
c. Phenytoin
d. DC cardioversion
199. Which of the following hypolipidemic drug acts by
inhibition of PCSK-9?
a. Atorvastatin
b. Evolocumab
c. Ezetimibe
d. Lomitapide
200. A patient presents with pituitary tumor that
overproduces growth hormone. surgical removal of the
tumor was incomplete. What is the first line treatment of
this patient?
a. Leuprolide
b. Octreotide
c. Nafarelin
d. Goserelin
201. A 40-year-old male presents with protrusion of chin,
excessive sweating, impaired glucose tolerance and enlargement
of hands and feet? Which of the following drugs is a growth
hormone receptor antagonist used to treat this condition?
a. Pegvisomant
b. Octreotide
c. Cabergoline
d. Olcegepant
202. A 40-year-old diabetic female presented to emergency with
abdominal pain, vomiting and recent onset confusion. On examination, she
had irregular breathing and dehydration. Her blood sugar is 539 mg/dl and
there was presence of ketone bodies in the urine. Blood pressure of the
patient is 80/50 mm Hg. What is the next best step in the management of
this patient?
a. Regular insulin
b. Intravenous fluids
c. Intravenous fluids with regular insulin
d. Long acting insulin
203. A female presented with galactorrhea. Her urine pregnancy
test was negative. MRI of head revealed a large pituitary tumor.
Patient refused to undergo surgery for the tumor. Which of the
following is the best drug for the treatment of this patient?
a. Octreotide
b. Bromocriptine
c. Promethazine
d. Clozapine
204. A patient with opioid poisoning presents with severe
respiratory depression. What is the most effective drug for
treatment of this patient?
a. Fomepizole
b. Naltrexone
c. Flumazenil
d. Naloxone
205. A patient on anti-depressant therapy presents with
elevated body temperature, dilated pupil, palpitations and low
blood pressure. ECG shows tachycardia, broad QRS complex and
right axis deviation. Which of the following interventions must
be done immediately?
a. Wait and watch
b. Intravenous sodium bicarbonate
c. Intravenous esmolol
d. DC cardioversion
206. A patient presents with tremors, rigidity and
bradykinesia. Which of the following drugs can be used for
the treatment of this patient?
a. Selegiline
b. Donepezil
c. Fluoxetine
d. Haloperidol
207. A child born by normal vaginal delivery developed repeated
attacks of flexion of neck over the trunk and jerks in the hands.
EEG shows the presence of hypsarrhythmia. Anti-epileptic drug
of choice for this patient is?
a. Phenobarbitone
b. Phenytoin
c. ACTH
d. Levetiracetam
208. An 11-year-old boy presented to emergency with vomiting.
Parents gave history of consuming 10-15 tablets of ferrous sulphate
a day before. What is the antidote of iron for treatment of this
patient?
a. Dimercaprol
b. Desferrioxamine
c. d- Penicillamine
d. Activated charcoal
209. A deep vein thrombosis patient was started on an
anticoagulant therapy. Next day, the patient presented with the
features shown in the diagram below. Likely drug implicated for
this adverse effect is
a. Heparin
b. Rivaroxaban
c. Warfarin
d. Dabigatran
210. A patient with COPD was given intravenous aminophylline
therapy following which the patient developed diuresis. This is
due to action on which effector?
a. Adenosine receptors
b. Beta 2 adrenergic receptors
c. IL-10
d. Histone deacetylase
211. A female with ovarian cancer was on cisplatin therapy. He
presented to hospital with several episodes of vomiting. After
giving an antiemetic drug, the patient developed dystonia. The
anti-emetic drug most likely to be responsible for these features is
a. Metoclopramide
b. Meclizine
c. Ondansetron
d. Scopolamine
212. A pregnant female with urinary tract infection took
antimicrobials for the same. The baby of this female developed
tendon rupture and arthropathy. What is the likely mechanism of
action of the antimicrobial consumed by the pregnant female?
a.Folic acid synthesis inhibitor
b.Mycolic acid synthesis inhibitor
c. DNA gyrase inhibitor
d.DNA inhibitor
213. A 48 year old patient presented to OPD with tingling and
numbness of fingers. The patient is chronic alcoholic and was
recently started on antitubercular treatment. What should be the
treatment of this patient?
a. Vitamin B3
b. Vitamin B6
c. Vitamin B1
d. Vitamin B12
214. In MDR tuberculosis, which of following drug combinations
show resistance?
a. Rifampicin and Fluoroquinolones
b. Isoniazid and rifampicin
c. Isoniazid and Pyrazinamide
d. Rifampicin and kanamycin
215. A patient presented with hyperkalemia. Which of the
following drugs can cause intracellular movement of and thus
decrease in extracellular potassium levels?
a. Atropine
b. Lactic acid
c. Epinephrine
d. Glucagon
216. Oral Factor Xa Inhibitor is:
a. Dabigatran etexilate
b. Rivaroxaban
c. Fondaparinux
d. Bivalirudin
217. High plasma protein binding of a drug results in:
a. Decreased glomerular filtration
b. High volume of distribution
c. Lowers duration of action
d. Less drug interaction
218. DOC for prophylaxis against Diphtheria:
a. Erythromycin
b. Rifampicin
c. Cloxacillin
d. Ciprofloxacin
219. Variation in drug responsiveness to a particular dose of the drug in
different individuals can be obtained from:
a. Graded Dose Response Curve
b. Efficacy
c. Potency
d. Quantal Dose Response Curve
220. Most specific antiemetic for Chemotherapy induced vomiting is:
a. Doxylamine
b. Tegaserod
c. Granisetron
d. Domperidone
221. Identify the true statement regarding Clinical Trials:
a. Phase 1 is done to determine efficacy in patients
b. Healthy volunteers are recruited for the first time in Phase II
c. Randomized Controlled Trials in patients is done in Phase III
d. Phase IV is Pharmacokinetics study in animals
222. Fluoroquinolone with highest oral bioavailability:
a. Levofloxacin
b. Gemifloxacin
c. Ciprofloxacin
d. Norfloxacin
223. Which of the following statement is true regarding Fixed Dose
Combination of drugs?
A. Adverse effect of one drug may be reduced by the other drug
B. Two drugs with different pharmacokinetics can be combined easily
C. Dose of one drug can be altered independently as per requirement
D. Adverse effect can be ascribed to a single drug
224. Which of the following is a K+ Channel Opener?
A. Ranolazine
B. Nicorandil
C. Verapamil
D. Quinidine
225. Tolvaptan is used for:
A. SIADH
B. Central DI
C. Von Willebrand Disease
D. Catecholamine resistant Shock
226. Mechanism of action of Oseltamivir:
A. DNA polymerase inhibition
B. Inhibition of viral mRNA
C. Blocking viral uncoating
D. Neuraminidase inhibition
227. Buspirone acts on:
A. 5HT1A
B. 5HT1B
C. 5HT2
D. 5HT3
228. Carbapenem with maximum seizure risk is:
A. Imipenem
B. Meropenem
C. Ertapenem
D. Doripenem
229. Cilastatin is given in combination with Imipenem because:
A. Cilastatin prevents degradation of Imipenem in kidney
B. Cilastatin increases absorption of Imipenem
C. Inhibits the enzymes that digest Imipenem in stomach
D. Reduces side effects of Imipenem.
230. Mechanism of action of Triazoles:
A. Inhibits ergosterol biosynthesis
B. Inhibits tubulin
C. Inhibits glucan synthesis
D. Inhibits cell wall synthesis
231. Identify the false statement regarding Teduglutide is:
A. Used for Short-Bowel Syndrome
B. GLP2 agonist
C. A pancreatic enzyme
D. A gut Hormone
232. Hypertension and pulmonary edema associated with scorpion sting is
managed by:
A. Carvedilol
B. Prazosin
C. Spironolactone
D. Phentolamine
233. Mannitol is used in the management of:
A. Congestive Cardiac Failure
B. Acute Congestive Glaucoma
C. Acute Renal Failure
D. Pulmonary edema
234. Drug inhibiting granulocyte migration is:
A. Montelukast
B. Cromoglycate
C. Colchicine
D. Felbamate
235. Drug capable of causing ocular hypotension with apnoea in infants is:
A. Acetazolamide
B. Latanoprost
C. Brimonidine
D. Apraclonidine
236. Inhalational anesthetic with highest respiratory irritation is:
A. Desflurane
B. Nitrous Oxide
C. Sevoflurane
D. Halothane
237. Drug used for smoking cessation:
A. Varenicline
B. Acamprosate
C. Nalmefene
D. Gabapentin
238. Indication for use of Pegylated Filgrastim is in the correction of:
A. Anaemia
B. Neutropenia
C. Thrombocytopenia
D. Pancytopenia
239. Agent used for eliciting diagnostic differentiation of Myasthenia
Gravis from Cholinergic crisis is:
A. Ecothiophate
B. Edrophonium
C. Neostigmine
D. Ambenonium
240. Anesthetic not painful on Intravenous administration is:
A. Ketamine
B. Propofol
C. Etomidate
D. Methohexital
241. Which IV anesthetic does not cause cardiac depression?
A. Etomidate
B. Ketamine
C. Methohexital
D. Thiopentone
242. Mechanism of action of curare like drugs:
A. Competitive, Non depolarizing block at the Nm cholinergic receptors
B. Noncompetitive, Non depolarizing block at the Nm cholinergic receptors
C. Competitive, Depolarizing block at the Nm cholinergic receptors
D. Noncompetitive, Depolarizing block at the Nm cholinergic receptors
243. Reason for preferring cis-Atracurium over Atracurium is:
A. Equal potency like Atracurium
B. Lesser provocation of histamine release
C. Short fast action
D. Does not undergo Hoffman elimination
244. Drug of Choice for Digoxin induced Ventricular Tachycardia:
A. Propranolol
B. Diltiazem
C. Verapamil
D. Lignocaine
245. Contraindications to use of Beta Blockers:
A. Glaucoma
B. Tachycardia
C. Bronchial asthma
D. Hypertension
246. Drug acting via tyrosine kinase receptor is
A. TRH
B. TSH
C. Insulin
D. MSH
247. A patient on lithium therapy developed hypertension. He was started
on thiazides for hypertension. After few days, he developed coarse tremors
and other symptoms suggestive of lithium toxicity. Explain the likely
mechanism of this interaction.
A. Thiazide inhibits the metabolism of lithium
B. Thiazides act as an add on drug to lithium
C. Thiazides increase the tubular reabsorption of lithium
D. Thiazides cause loss of water thereby increased lithium levels.
248. Which of the following drugs shown in the graph below has highest
potency?
A. Drug A
B. Drug B
C. Drug C
D. Both Drug A and B
249. Mechanism of resistance to penicillins via beta lactamase is
A. Altered penicillin binding proteins
B. Drug efflux
C. Breaks drug structure
D. Alteration in 50S ribosome structure
250. Which of the following drugs act by inhibiting DNA replication?
A. 6 Mercaptopurine
B. Actinomycin D
C. Mitomycin C
D. Asparaginase
251. A female patient was on lithium therapy for bipolar disorder for 6
months. She kept the fast for few days due to religious reasons and
presented with seizures, coarse tremors, confusion and weakness of limbs.
Which of the following should be done to diagnose her condition?
A. Serum Electrolytes
B. Serum lithium levels
C. ECG
D. MRI
252. Which of the following drugs is used as nail lacquer for fungal
infections
A. Fluconazole
B. Nystatin
C. Itraconazole
D. Terbinafine
253. Pegloticase is used for treatment of :-
A. Ankylosing spondylosis
B. CPPD
C. Chronic tophaceous gout
D. Refractory Rheumatoid arthritis
254. Which of the following is not a prokinetic?
A. 5HT4 agonist
B. D2 blocker
C. Macrolides
D. Diphenoxymethane
255. Fluoroquinolone contraindicated in liver disease is
A. Levofloxain
B. Pefloxacin
C. Ofloxacin
D. Lomefloxacin
256. At a high altitude of 3000m, a person complains of breathlessness. All
of the following can be used for management of this person except
A. Intravenous digoxin
B. Oxygen supplementation
C. Immediate descent
D. Acetazolamide
257. True about non-competitive inhibition is
A. Km remains same, Vmax decreases
B. Km increases, Vmax remains same
C. Km decreases, Vmax increases
D. Km increases, Vmax increases
258. Theophylline causes diuresis because of
A. PDE inhibition
B. Adenosine A1 receptor antagonism
C. Beta 2 agonism
D. M3 receptor antagonism
259. A patient with diabetes developed post-operative urinary retention.
Which of the following drugs can be used for short term treatment to relieve
the symptoms of this person?
A. Bethanechol
B. Methacholine
C. Terazosin
D. Tamsulosin
260. Drug of choice for invasive aspergillosis is
A. Posaconazole
B. Voriconazole
C. Liposomal AMB
D. Caspofungin
261. Which of the following antihypertensive drug is avoided in patients
with high serum uric acid levels?
A. Hydrochlorothiazide
B. Enalapril
C. Prazosin
D. Atenolol
262. Which of the following drug acts by inhibiting the transcription of
DNA to RNA?
A. Rifampicin
B. Nitrofurantoin
C. Ciprofloxacin
D. Novobiocin
263. A patient was recently started on Fluphenazine. Few weeks later, he
developed tremors, rigidity, bradykinesia and excessive salivation. First line
of management for this patient is
A. Selegiline
B. Trihexyphenidyl
C. Pramipexole
D. Amantadine
264. Which of the following antimicrobials should not be given to a chronic
asthmatic patient managed on theophylline therapy?
A. Erythromycin
B. Cefotaxime
C. Cotrimoxazole
D. Amoxicillin
265. A person was given a muscle relaxant that competitively blocks
nicotinic receptors. Which of the following drug is used for reversal of
muscle relaxation after surgery?
A. Neostigmine
B. Carbachol
C. Succinylcholine
D. Physostigmine
266. Anti-glaucoma drug that acts by increasing uveoscleral outlfow is
A. Latanoprost
B. Timolol
C. Pilocarpine
D. Dorzolamide
267. A patient diagnosed with Rheumatoid arthritis was on medications.
After 2 years, he developed blurring of vision and was found to have
corneal opacity. Which drug is most likely to cause this?
A. Sulfasalazine
B. Chloroquine
C. Methotrexate
D. Leflunomide
268. A boy is planning to travel by bus. Which of the following drug can be
used to prevent motion sickness in this person?
A. Promethazine
B. Cetrizine
C. Loratidine
D. Fexofenadine
269. Which of the following antiarrhythmic drug is contraindicated in a
patient with interstitial lung disease?
A. Amiodarone
B. Sotalol
C. Quinidine
D. Lignocaine
270. Which of the following is a late inward sodium channel blocker?
A. Ivabradine
B. Ranolazine
C. Trimetazidine
D. Fasudil
271. A patient of biliary colic presented to hospital. Intern gave an injection
and the pain worsened. Which is the most likely injection given?
A. Morphine
B. Diclofenac
C. Nefopam
D. Etoricoxib
272. Drug of choice for post menopausal osteoporosis is
A. Estrogen
B. Bisphosphonates
C. Teriparatide
D. Thyroxine
273. Which of the following is not used as emergency contraceptive?
A. Danazol
B. Levonorgestrel
C. Mifepristone
D. IUCD
274. As an Indian medical intern, which of the following is the correct
format for prescription of alprazolam?
a. Tablet alprazolam 0.5 mg once a day before bed time for 7 days
b. Tablet alprazolam 0.5 mg HS for 7 days
c. Tablet alprazolam 500 mcg one tablet OD for 7 days
d. Tablet alprazolam ½ mg tablet HS daily
275. Rate of administration of a drug is equal to rate of elimination. How
will you calculate the dosing rate of the drug to maintain steady state
concentration?
a. Dosing rate = Vd x target plasma concentration
b. Dosing rate = CL x target plasma concentration
c. Dosing rate = Vd/target plasma concentration
d. Dosing rate = CL/target plasma concentration
276. A patient with bronchial asthma presents with raised intraocular
pressure. Treatment of open angle glaucoma in this patient is
a. Latanoprost
b. Alprostadil
c. Gemeprost
d. Carboprost
277. A patient presented with suspected cocaine overdose. Which of th
following features is not seen in this patient?
a. Bradycardia
b. Agitation
c. Myocardial infarction
d. Hyperthermia
278. A child presents to hospital with agitation, photophobia and retention
of urine. History reveals that he has eaten a wild fruit given by his friend.
Which of the following substances is the likely cause of poisoning and its
antidote respectively?
a. Dhatura, Physostigmine
b. Dhatura, Pralidoxime
c. Yellow Oleander, Physostigmine
d. Yellow Oleander, Pralidoxime
279. A 45-year-old patient being treated with low dose aspirin since 6
months presented with rectal bleeding. Inhibition of which of the following
substance is likely to responsible for the bleeding?
a. TXA2
b. LT
c. Bradykinin
d. PGI2
280. A 34-week pregnant female with polyhydramnios presents with
labour pain. She was treated with indomethacin earlier. Which of the
following can be likely outcome in the baby if delivery occurs at this time?
a. Patent ductus arteriosus
b. Premature closure of ductus arteriosus
c. Patent ductus venosus
d. Premature closure of ductus venosus
281. A 60-year-old female with renal disease was admitted for
pyelolithotomy. Post operative analgesic of choice in this patient is?
a. Diclofenac
b. Indomethacin
c. Naproxen
d. Acetaminophen
282. A 45-year-old patient presented with the symptoms shown in image
below. Patient is a known hypertensive and was taking some
antihypertensive drug. Which of the following drug is the likely cause of this
condition?
a. Lisinopril
b. Clonidine
c. Nifedipine
d. Timolol
283. Tocilizumab is an immunosuppressive drug used for Rheumatoid
arthritis. It is a monoclonal antibody against
a. IL-6
b. IL-12
c. IL-2
d. IL-3
284. A patient on digoxin therapy and had a level of 1 ng/ml. He was
receiving several other drugs and presented 2 months later with flattening
of T waves in ECG. Now the plasma level of digoxin is 3.8 ng/ml. Which of
the following drug is likely to be responsible?
a. Triamterene
b. Atenolol
c. Clarithromycin
d. KCl
285. A 26-year-old female came to hospital for pre-conceptional
counselling. Patient is a known hypertensive compliant to drug therapy.
Which of the following drugs will you advise to stop?
a. Methyl dopa
b. Labetalol
c. Lisinopril
d. Nifedipine
286. A 60-year-old post-menopausal woman having a previous history of
Colle’s fracture presented with backache. Which of the following statements
about anti-osteoporosis drugs is false regarding this patient?
a. Calcium requirement is 1200 mg per day
b. Oral vitamin D3 should be given along with calcium supplements
c. Bisphosphonates therapy should not be given for more than one year
d. Teriparatide therapy should be followed by bisphosphonate therapy
287. An anesthetist injected bupivacaine in a patient for axillary nerve
block. Later, the patient developed restlessness, agitation and
cardiovascular collapse. Next best step in the management would be?
a. Cardiac resuscitation + dopamine
b. Cardiac resuscitation + dantrolene
c. Cardiac resuscitation + sodium bicarbonate
d. Cardiac resuscitation + 20 % intralipid
288. A patient was admitted for surgery and halothane is being planned to
be used as an anaesthetic agent. The patient was explained about the
adverse effect of malignant hyperthermia with this drug. Which of the
following drugs may also result in malignant hyperthermia?
a. Succinylcholine
b. D-Tubocurarine
c. Dantrolene
d. Baclofen
289. A patient presented to hospital with three bouts of vomiting and
treated with anti-emetic drug. Vomiting subsided but after sometime, he
developed abnormal movements. Which of the following drug is used for
treatment of these motor symptoms?
a. Benzhexol
b. Cyproheptadine
c. L-dopa
d. Hyoscine
290. A female taking oral contraceptives acquired tuberculosis. After
prescribing anti-tubercular therapy, physician advised the patient for
alternative contraception. What is the probable reason of this advise?
a. Rifampicin causes teratogenicity
b. Isoniazid is teratogenic
c. Rifampicin increases the metabolism of oral contraceptives
d. Oral contraceptives decrease the efficacy of anti-tubercular therapy
291. Metronidazole is used for treatment of various anaerobic infections as
well as bacterial vaginosis. Patient is instructed to avoid taking which of the
following substance after metronidazole prescription to avoid possible
adverse effects?
a. Grapefruit juice
b. Milk
c. Alcohol
d. Orange juice
292. A patient of borderline leprosy received multi drug therapy (MDT). He
developed skin lesions as shown in the figure below. How should this patient
be managed?
a. Stop MDT and give thalidomide
b. Continue MDT and give thalidomide
c. Stop MDT and give steroids
d. Continue MDT and give steroids
293. A 20-year-old boy presented with slow growing mass on the medial
side of thigh. Investigations revealed it to be osteosarcoma and patient
was started on methotrexate. Mechanism of action of methotrexate as an
anticancer drug is
a. Inhibition of Dihydrofolate reductase
b. Inhibition of N5 tetrahydro folate reductase
c. Inhibition of tyrosine kinase
d. Inhibition of purine metabolism
294. Topical application of 400 mcg/ml mitomycin C is used for
a. Subglottic stenosis
b. Tympanoplasty
c. Rhinocerebral mucormycosis
d. Nasopharyngeal bleeding
THANK YOU!

More Related Content

Similar to PYQ FOR ORR.pptx

Krok 1 2014 - pharmacology
Krok 1   2014 - pharmacologyKrok 1   2014 - pharmacology
Krok 1 2014 - pharmacology
Eneutron
 
Pharmacogy
PharmacogyPharmacogy
Pharmacogy
guest13d8350
 
MCQ ANS 30.pptx
MCQ ANS 30.pptxMCQ ANS 30.pptx
MCQ ANS 30.pptx
pharmacologycmccbe
 
Nepal Pharmacy Council License Exam Model set II (Pharmacist)
Nepal Pharmacy Council License Exam Model set II  (Pharmacist)Nepal Pharmacy Council License Exam Model set II  (Pharmacist)
Nepal Pharmacy Council License Exam Model set II (Pharmacist)
Nabin Bist
 
Block G pre proff KGMC 2022 print with key.pdf
Block G pre proff KGMC 2022 print with key.pdfBlock G pre proff KGMC 2022 print with key.pdf
Block G pre proff KGMC 2022 print with key.pdf
Khan676722
 
UVAS clinical pharmacy part 1 mcqs .pdf
UVAS clinical pharmacy part 1 mcqs  .pdfUVAS clinical pharmacy part 1 mcqs  .pdf
UVAS clinical pharmacy part 1 mcqs .pdf
UVAS
 
CVS_Hormones_MCQs.pptx
CVS_Hormones_MCQs.pptxCVS_Hormones_MCQs.pptx
CVS_Hormones_MCQs.pptx
ShreevarshniLakshmik
 
NEET PG MAY 2022
NEET PG MAY 2022NEET PG MAY 2022
NEET PG MAY 2022
Shivankan Kakkar
 
New microsoft office word document (2)
New microsoft office word document (2)New microsoft office word document (2)
New microsoft office word document (2)
Rajesh Mandade
 
Krok 1 - 2010 Question Paper (General medicine)
Krok 1 - 2010 Question Paper (General medicine)Krok 1 - 2010 Question Paper (General medicine)
Krok 1 - 2010 Question Paper (General medicine)
Eneutron
 
Clinical Problems.ppt.pptx
Clinical Problems.ppt.pptxClinical Problems.ppt.pptx
Clinical Problems.ppt.pptx
NirmalaD21
 
Med test collaborative
Med test collaborativeMed test collaborative
Med test collaborative
Kim Uddo
 
Krok 1 - 2013 Question Paper (General medicine)
Krok 1 - 2013 Question Paper (General medicine)Krok 1 - 2013 Question Paper (General medicine)
Krok 1 - 2013 Question Paper (General medicine)
Eneutron
 
Anticholinergics
Anticholinergics  Anticholinergics
Anticholinergics
irinepetrescu
 
Krok 1 - 2013 Question Paper (Pharmacy)
Krok 1 - 2013 Question Paper (Pharmacy)Krok 1 - 2013 Question Paper (Pharmacy)
Krok 1 - 2013 Question Paper (Pharmacy)
Eneutron
 
Pharmacology MCQ Free PDF Objective Question Answer.pptx
Pharmacology MCQ Free PDF Objective Question Answer.pptxPharmacology MCQ Free PDF Objective Question Answer.pptx
Pharmacology MCQ Free PDF Objective Question Answer.pptx
Zawarali786
 
Quiz%20on%20diabetes%20Questions.pptx
Quiz%20on%20diabetes%20Questions.pptxQuiz%20on%20diabetes%20Questions.pptx
Quiz%20on%20diabetes%20Questions.pptx
Rajesh Kumar
 
Anesthesia complications mcqs by israr ysfz
Anesthesia complications mcqs by israr ysfzAnesthesia complications mcqs by israr ysfz
Anesthesia complications mcqs by israr ysfz
israr khan
 
Ejemplo examen fármacos en Cardiología .ppt
Ejemplo examen  fármacos en  Cardiología .pptEjemplo examen  fármacos en  Cardiología .ppt
Ejemplo examen fármacos en Cardiología .ppt
JacobMush
 
ETAS_MCQ_16 dermatological drugs
ETAS_MCQ_16 dermatological drugsETAS_MCQ_16 dermatological drugs
ETAS_MCQ_16 dermatological drugs
Derma202
 

Similar to PYQ FOR ORR.pptx (20)

Krok 1 2014 - pharmacology
Krok 1   2014 - pharmacologyKrok 1   2014 - pharmacology
Krok 1 2014 - pharmacology
 
Pharmacogy
PharmacogyPharmacogy
Pharmacogy
 
MCQ ANS 30.pptx
MCQ ANS 30.pptxMCQ ANS 30.pptx
MCQ ANS 30.pptx
 
Nepal Pharmacy Council License Exam Model set II (Pharmacist)
Nepal Pharmacy Council License Exam Model set II  (Pharmacist)Nepal Pharmacy Council License Exam Model set II  (Pharmacist)
Nepal Pharmacy Council License Exam Model set II (Pharmacist)
 
Block G pre proff KGMC 2022 print with key.pdf
Block G pre proff KGMC 2022 print with key.pdfBlock G pre proff KGMC 2022 print with key.pdf
Block G pre proff KGMC 2022 print with key.pdf
 
UVAS clinical pharmacy part 1 mcqs .pdf
UVAS clinical pharmacy part 1 mcqs  .pdfUVAS clinical pharmacy part 1 mcqs  .pdf
UVAS clinical pharmacy part 1 mcqs .pdf
 
CVS_Hormones_MCQs.pptx
CVS_Hormones_MCQs.pptxCVS_Hormones_MCQs.pptx
CVS_Hormones_MCQs.pptx
 
NEET PG MAY 2022
NEET PG MAY 2022NEET PG MAY 2022
NEET PG MAY 2022
 
New microsoft office word document (2)
New microsoft office word document (2)New microsoft office word document (2)
New microsoft office word document (2)
 
Krok 1 - 2010 Question Paper (General medicine)
Krok 1 - 2010 Question Paper (General medicine)Krok 1 - 2010 Question Paper (General medicine)
Krok 1 - 2010 Question Paper (General medicine)
 
Clinical Problems.ppt.pptx
Clinical Problems.ppt.pptxClinical Problems.ppt.pptx
Clinical Problems.ppt.pptx
 
Med test collaborative
Med test collaborativeMed test collaborative
Med test collaborative
 
Krok 1 - 2013 Question Paper (General medicine)
Krok 1 - 2013 Question Paper (General medicine)Krok 1 - 2013 Question Paper (General medicine)
Krok 1 - 2013 Question Paper (General medicine)
 
Anticholinergics
Anticholinergics  Anticholinergics
Anticholinergics
 
Krok 1 - 2013 Question Paper (Pharmacy)
Krok 1 - 2013 Question Paper (Pharmacy)Krok 1 - 2013 Question Paper (Pharmacy)
Krok 1 - 2013 Question Paper (Pharmacy)
 
Pharmacology MCQ Free PDF Objective Question Answer.pptx
Pharmacology MCQ Free PDF Objective Question Answer.pptxPharmacology MCQ Free PDF Objective Question Answer.pptx
Pharmacology MCQ Free PDF Objective Question Answer.pptx
 
Quiz%20on%20diabetes%20Questions.pptx
Quiz%20on%20diabetes%20Questions.pptxQuiz%20on%20diabetes%20Questions.pptx
Quiz%20on%20diabetes%20Questions.pptx
 
Anesthesia complications mcqs by israr ysfz
Anesthesia complications mcqs by israr ysfzAnesthesia complications mcqs by israr ysfz
Anesthesia complications mcqs by israr ysfz
 
Ejemplo examen fármacos en Cardiología .ppt
Ejemplo examen  fármacos en  Cardiología .pptEjemplo examen  fármacos en  Cardiología .ppt
Ejemplo examen fármacos en Cardiología .ppt
 
ETAS_MCQ_16 dermatological drugs
ETAS_MCQ_16 dermatological drugsETAS_MCQ_16 dermatological drugs
ETAS_MCQ_16 dermatological drugs
 

Recently uploaded

Topic: SICKLE CELL DISEASE IN CHILDREN-3.pdf
Topic: SICKLE CELL DISEASE IN CHILDREN-3.pdfTopic: SICKLE CELL DISEASE IN CHILDREN-3.pdf
Topic: SICKLE CELL DISEASE IN CHILDREN-3.pdf
TinyAnderson
 
EWOCS-I: The catalog of X-ray sources in Westerlund 1 from the Extended Weste...
EWOCS-I: The catalog of X-ray sources in Westerlund 1 from the Extended Weste...EWOCS-I: The catalog of X-ray sources in Westerlund 1 from the Extended Weste...
EWOCS-I: The catalog of X-ray sources in Westerlund 1 from the Extended Weste...
Sérgio Sacani
 
bordetella pertussis.................................ppt
bordetella pertussis.................................pptbordetella pertussis.................................ppt
bordetella pertussis.................................ppt
kejapriya1
 
ANAMOLOUS SECONDARY GROWTH IN DICOT ROOTS.pptx
ANAMOLOUS SECONDARY GROWTH IN DICOT ROOTS.pptxANAMOLOUS SECONDARY GROWTH IN DICOT ROOTS.pptx
ANAMOLOUS SECONDARY GROWTH IN DICOT ROOTS.pptx
RASHMI M G
 
Remote Sensing and Computational, Evolutionary, Supercomputing, and Intellige...
Remote Sensing and Computational, Evolutionary, Supercomputing, and Intellige...Remote Sensing and Computational, Evolutionary, Supercomputing, and Intellige...
Remote Sensing and Computational, Evolutionary, Supercomputing, and Intellige...
University of Maribor
 
Travis Hills' Endeavors in Minnesota: Fostering Environmental and Economic Pr...
Travis Hills' Endeavors in Minnesota: Fostering Environmental and Economic Pr...Travis Hills' Endeavors in Minnesota: Fostering Environmental and Economic Pr...
Travis Hills' Endeavors in Minnesota: Fostering Environmental and Economic Pr...
Travis Hills MN
 
BREEDING METHODS FOR DISEASE RESISTANCE.pptx
BREEDING METHODS FOR DISEASE RESISTANCE.pptxBREEDING METHODS FOR DISEASE RESISTANCE.pptx
BREEDING METHODS FOR DISEASE RESISTANCE.pptx
RASHMI M G
 
NuGOweek 2024 Ghent programme overview flyer
NuGOweek 2024 Ghent programme overview flyerNuGOweek 2024 Ghent programme overview flyer
NuGOweek 2024 Ghent programme overview flyer
pablovgd
 
Nucleic Acid-its structural and functional complexity.
Nucleic Acid-its structural and functional complexity.Nucleic Acid-its structural and functional complexity.
Nucleic Acid-its structural and functional complexity.
Nistarini College, Purulia (W.B) India
 
Applied Science: Thermodynamics, Laws & Methodology.pdf
Applied Science: Thermodynamics, Laws & Methodology.pdfApplied Science: Thermodynamics, Laws & Methodology.pdf
Applied Science: Thermodynamics, Laws & Methodology.pdf
University of Hertfordshire
 
ESR spectroscopy in liquid food and beverages.pptx
ESR spectroscopy in liquid food and beverages.pptxESR spectroscopy in liquid food and beverages.pptx
ESR spectroscopy in liquid food and beverages.pptx
PRIYANKA PATEL
 
Shallowest Oil Discovery of Turkiye.pptx
Shallowest Oil Discovery of Turkiye.pptxShallowest Oil Discovery of Turkiye.pptx
Shallowest Oil Discovery of Turkiye.pptx
Gokturk Mehmet Dilci
 
Cytokines and their role in immune regulation.pptx
Cytokines and their role in immune regulation.pptxCytokines and their role in immune regulation.pptx
Cytokines and their role in immune regulation.pptx
Hitesh Sikarwar
 
8.Isolation of pure cultures and preservation of cultures.pdf
8.Isolation of pure cultures and preservation of cultures.pdf8.Isolation of pure cultures and preservation of cultures.pdf
8.Isolation of pure cultures and preservation of cultures.pdf
by6843629
 
Oedema_types_causes_pathophysiology.pptx
Oedema_types_causes_pathophysiology.pptxOedema_types_causes_pathophysiology.pptx
Oedema_types_causes_pathophysiology.pptx
muralinath2
 
mô tả các thí nghiệm về đánh giá tác động dòng khí hóa sau đốt
mô tả các thí nghiệm về đánh giá tác động dòng khí hóa sau đốtmô tả các thí nghiệm về đánh giá tác động dòng khí hóa sau đốt
mô tả các thí nghiệm về đánh giá tác động dòng khí hóa sau đốt
HongcNguyn6
 
Phenomics assisted breeding in crop improvement
Phenomics assisted breeding in crop improvementPhenomics assisted breeding in crop improvement
Phenomics assisted breeding in crop improvement
IshaGoswami9
 
Micronuclei test.M.sc.zoology.fisheries.
Micronuclei test.M.sc.zoology.fisheries.Micronuclei test.M.sc.zoology.fisheries.
Micronuclei test.M.sc.zoology.fisheries.
Aditi Bajpai
 
Randomised Optimisation Algorithms in DAPHNE
Randomised Optimisation Algorithms in DAPHNERandomised Optimisation Algorithms in DAPHNE
Randomised Optimisation Algorithms in DAPHNE
University of Maribor
 
The binding of cosmological structures by massless topological defects
The binding of cosmological structures by massless topological defectsThe binding of cosmological structures by massless topological defects
The binding of cosmological structures by massless topological defects
Sérgio Sacani
 

Recently uploaded (20)

Topic: SICKLE CELL DISEASE IN CHILDREN-3.pdf
Topic: SICKLE CELL DISEASE IN CHILDREN-3.pdfTopic: SICKLE CELL DISEASE IN CHILDREN-3.pdf
Topic: SICKLE CELL DISEASE IN CHILDREN-3.pdf
 
EWOCS-I: The catalog of X-ray sources in Westerlund 1 from the Extended Weste...
EWOCS-I: The catalog of X-ray sources in Westerlund 1 from the Extended Weste...EWOCS-I: The catalog of X-ray sources in Westerlund 1 from the Extended Weste...
EWOCS-I: The catalog of X-ray sources in Westerlund 1 from the Extended Weste...
 
bordetella pertussis.................................ppt
bordetella pertussis.................................pptbordetella pertussis.................................ppt
bordetella pertussis.................................ppt
 
ANAMOLOUS SECONDARY GROWTH IN DICOT ROOTS.pptx
ANAMOLOUS SECONDARY GROWTH IN DICOT ROOTS.pptxANAMOLOUS SECONDARY GROWTH IN DICOT ROOTS.pptx
ANAMOLOUS SECONDARY GROWTH IN DICOT ROOTS.pptx
 
Remote Sensing and Computational, Evolutionary, Supercomputing, and Intellige...
Remote Sensing and Computational, Evolutionary, Supercomputing, and Intellige...Remote Sensing and Computational, Evolutionary, Supercomputing, and Intellige...
Remote Sensing and Computational, Evolutionary, Supercomputing, and Intellige...
 
Travis Hills' Endeavors in Minnesota: Fostering Environmental and Economic Pr...
Travis Hills' Endeavors in Minnesota: Fostering Environmental and Economic Pr...Travis Hills' Endeavors in Minnesota: Fostering Environmental and Economic Pr...
Travis Hills' Endeavors in Minnesota: Fostering Environmental and Economic Pr...
 
BREEDING METHODS FOR DISEASE RESISTANCE.pptx
BREEDING METHODS FOR DISEASE RESISTANCE.pptxBREEDING METHODS FOR DISEASE RESISTANCE.pptx
BREEDING METHODS FOR DISEASE RESISTANCE.pptx
 
NuGOweek 2024 Ghent programme overview flyer
NuGOweek 2024 Ghent programme overview flyerNuGOweek 2024 Ghent programme overview flyer
NuGOweek 2024 Ghent programme overview flyer
 
Nucleic Acid-its structural and functional complexity.
Nucleic Acid-its structural and functional complexity.Nucleic Acid-its structural and functional complexity.
Nucleic Acid-its structural and functional complexity.
 
Applied Science: Thermodynamics, Laws & Methodology.pdf
Applied Science: Thermodynamics, Laws & Methodology.pdfApplied Science: Thermodynamics, Laws & Methodology.pdf
Applied Science: Thermodynamics, Laws & Methodology.pdf
 
ESR spectroscopy in liquid food and beverages.pptx
ESR spectroscopy in liquid food and beverages.pptxESR spectroscopy in liquid food and beverages.pptx
ESR spectroscopy in liquid food and beverages.pptx
 
Shallowest Oil Discovery of Turkiye.pptx
Shallowest Oil Discovery of Turkiye.pptxShallowest Oil Discovery of Turkiye.pptx
Shallowest Oil Discovery of Turkiye.pptx
 
Cytokines and their role in immune regulation.pptx
Cytokines and their role in immune regulation.pptxCytokines and their role in immune regulation.pptx
Cytokines and their role in immune regulation.pptx
 
8.Isolation of pure cultures and preservation of cultures.pdf
8.Isolation of pure cultures and preservation of cultures.pdf8.Isolation of pure cultures and preservation of cultures.pdf
8.Isolation of pure cultures and preservation of cultures.pdf
 
Oedema_types_causes_pathophysiology.pptx
Oedema_types_causes_pathophysiology.pptxOedema_types_causes_pathophysiology.pptx
Oedema_types_causes_pathophysiology.pptx
 
mô tả các thí nghiệm về đánh giá tác động dòng khí hóa sau đốt
mô tả các thí nghiệm về đánh giá tác động dòng khí hóa sau đốtmô tả các thí nghiệm về đánh giá tác động dòng khí hóa sau đốt
mô tả các thí nghiệm về đánh giá tác động dòng khí hóa sau đốt
 
Phenomics assisted breeding in crop improvement
Phenomics assisted breeding in crop improvementPhenomics assisted breeding in crop improvement
Phenomics assisted breeding in crop improvement
 
Micronuclei test.M.sc.zoology.fisheries.
Micronuclei test.M.sc.zoology.fisheries.Micronuclei test.M.sc.zoology.fisheries.
Micronuclei test.M.sc.zoology.fisheries.
 
Randomised Optimisation Algorithms in DAPHNE
Randomised Optimisation Algorithms in DAPHNERandomised Optimisation Algorithms in DAPHNE
Randomised Optimisation Algorithms in DAPHNE
 
The binding of cosmological structures by massless topological defects
The binding of cosmological structures by massless topological defectsThe binding of cosmological structures by massless topological defects
The binding of cosmological structures by massless topological defects
 

PYQ FOR ORR.pptx

  • 1.
  • 2. 1. A 70-year-old patient has diabetes mellitus and hypertension. He presents with late-stage chronic kidney disease. Which of the following anti-diabetic drugs require least dose modification in renal disease? A. Sitagliptin B. Vildagliptin C. Linagliptin D. Saxagliptin
  • 3. 2. Which of the following drug can cause the side-effect given in the picture? A. Bleomycin B. Mitomycin C C. Doxorubicin D. Actinomycin D
  • 4. 3. A patient was on metoprolol therapy for hypertension. He was also administered verapamil. Which of the following is likely to happen with above combination therapy? A. Torsades de pointes B. Ventricular tachycardia C. Ventricular fibrillation D. Bradycardia with AV block
  • 5. 4. All of the following are advantages of enteric coated tablets except A.It increases the half-life of the drug B. It protects acid labile drugs from gastric pH C. It increases the absorption of drugs that are preferentially absorbed distal to stomach. D.It protects stomach from irritant drugs
  • 6. 5. A psychotropic drug inhibiting the reuptake of serotonin is likely to cause which of the following adverse effect? A. Visual disturbance B. Constipation C. Dry mouth D. Sexual dysfunction
  • 7. 6. Which of the following is a second line anti-tubercular drug? A.Streptomycin B. Ethionamide C. Vancomycin D.Acyclovir
  • 8. 7. A female with unilateral breast cancer is on tamoxifen therapy. Which of the following adverse effect is associated with the use of tamoxifen? A. Carcinoma in opposite breast B. Endometrial carcinoma C. Ovarian carcinoma D. Chronic myeloid leukemia
  • 9. 8. Combined oral contraceptive pills act by all of the following mechanisms except? A.Inhibit implantation by bringing change in the uterus B. Inhibit ovulation C. Makes cervical mucus thick and hostile D.Increase gonadotropin secretion
  • 10. 9. Which of the following drug should be avoided in a patient on rosuvastatin therapy? A. Clarithromycin B. Oxycodone C. Rivaroxaban D. Adalimumab
  • 11. 10. All of the following are bactericidal drugs against Mycobacterium except A.Kanamycin B. Thioacetazone C. Rifapentine D.Isoniazid
  • 12. 11. A patient is on long term anticonvulsant therapy. There is progressive contractility of visual field. The likely anti-epileptic drug responsible for this adverse effect is A.Vigabatrin B. Ethosuximide C. Phenobarbital D.Levetiracetam
  • 13. 12. Which of the following drugs act through intracellular receptors? A.Thyroxin B. Glucagon C. Epinephrine D.Parathyroid hormone
  • 14. 13. Drug of choice for treatment of hairy cell leukemia is? A.Interferon alpha B. Fludarabine C. Pentostatin D.Cladribine
  • 15. 14. Which of the following drug can be used as an antidote for fibrinolytic overdose? A.Ethamsylate B. Alteplase C. Andexanet alpha D.Epsilon amino caproic acid
  • 16. 15. Which of the following action is expected on stimulation of muscarinic receptors? A. Erection B. Ejaculation C. Increased contraction of cardiac muscles D. Bronchodilation
  • 17. 16. All of the following drugs are indicated in the treatment of NSTEMI except A. Clopidogrel B. Aspirin C. Streptokinase D. Prasugrel
  • 18. 17. A 28 year old female with Graves’ disease was taking medication for hyperthyroidism during pregnancy. She delivered a child with congenital anomaly ‘aplasia cutis congenita’. Most likely drug implicated for this teratogenic effect is? A.Carbimazole B. Levo-thyroxine C. Methylthiouracil D.Liothyronine
  • 19. 18. Which of the following antifungal drugs possess additional anti- inflammatory and antipruritic activity? A.Terbinafine B. Sertaconazole C. Luliconazole D.Ketoconazole
  • 20. 19. A 30 year old patient was started on lithium carbonate for bipolar disorder. The nurse will gave instructions on adverse effects of the therapy. The patient should report back if the following features are seen: A.Palpitations, chest pain, raised blood pressure B. Sedation, constipation, vomiting C. Tinnitus, severe diarrhea, ataxia D.Fever, skin rash, bradycardia
  • 21. 20. For plasma level monitoring, lithium estimation is done? A.Immediately after the dose B. 8 hours after last dose C. 12 hours after last dose D.24 hours after last dose
  • 22. 21. Dicumarol antagonizes the action of vitamin K by A.Competitive inhibition B. Non-competitive inhibition C. Uncompetitive inhibition D.Suicide inhibition
  • 23. 22. Which of the following is not used in treatment of polycystic ovarian disease? A.Letrozole B. Clomiphene citrate C. Ulipristal D.Combined oral contraceptives
  • 24. 23. Most potent opioid is a. Fentanyl b. Morphine c. Pentazocine d. Pethidine
  • 25. 24. Mechanism of action of allopurinol is A. Xanthine oxidase inhibition B. Recombinant uricase C. Increase uric acid excretion D. Decrease chemotaxis
  • 26. 25. Which is the following drug causes postural hypotension commonly? A. Beta blocker B. Alpha blocker C. ACE inhibitor D. Angiotensin receptor blockers
  • 27. 26. Which of the following drug is a carbonic anhydrase inhibitor? A. Acetazolamide B. Hydrochlorthiazide C. Furosemide D. Mannitol
  • 28. 27. Which is most cardiotoxic anticancer drug? A. Anthracyclines B. Cyclophosphamide C. Imatinib D. Tamoxifen
  • 29. 28. All of the following drugs are used for prophylaxis of migraine except A. Propranolol B. Imipramine C. Flutamide D. Flunarizine
  • 30. 29. A farmer presented with pin point pupil and increase in secretions. Likely poisoning is A. Organophosphate poisoning B. Dhatura poisoning C. Opioid poisoning D. Atropine poisoning
  • 31. 30. Which of the following is an oral direct thrombin inhibitor? A. Dabigatran B. Rivaroxaban C. Warfarin D. Lepirudin
  • 32. 31. Which of the following drugs follow zero order kinetics? A. Phenytoin B. Aspirin C. Morphine D. Propranolol
  • 33. 32. A lady has taken medication of ameiobiasis infection. In a party she drank alcohol. She has nausea vomiting and dizziness. Which anti-amoebic drug could have lead to interaction with alcohol to produce these symptoms? A. Metronidazole B. Nitazoxanide C. Paromomycin D. Diloxanide
  • 34. 33. Which anti-asthma drug is avoided with Erythromycin? A. Salbutamol B. Theophylline C. Terbutaline D. Ipratropium
  • 35. 34. A patient after valve replacement will require follow up treatment with A. Warfarin B. ACE inhibitors C. Beta blockers D. Thiazides
  • 36. 35. Antidote for opioid poisoning is A. Naloxone B. Pethidine C. Flumazenil D. Physostigmine
  • 37. 36. Monoclonal antibody used in cancer is A. Rituximab B. Cisplatin C. 5-Fluorouracil D. Methotrexate
  • 38. 37. Find therapeutic index of a drug from the information given below in the graph A. 1 B. 2 C. 4 D. 8
  • 39. 38. A 34 weeks pregnant lady presented with seizures. Her blood pressure is 200/110 mm Hg. Which drugs should be used for the treatment of this patient? A. Oral Labetalol B. IV labetalol plus magnesium sulphate C. IV Lorazepam D. IV Enalapril
  • 40. 39. Hydrochlorothiazide works by inhibiting A. Na+ Cl- pump in DCT B. Na+ Cl- pump in PCT C. Na+ K+ 2 Cl- pump in descending limb of loop of Henle D. Na+ K+ 2 Cl- pump in ascending limb of loop of Henle
  • 41. 40. Drug of choice for hyperthyroidism in third trimester of pregnancy is A. Carbimazole B. Propylthyouracil C. Sodium iodide D. Radioactive iodine
  • 42. 41. Mechanism of action of exenatide in diabetes mellitus is A. It is analogue of GLP released from gut and increase glucose dependent insulin secretion B. It is DPP-4 inhibitor and result in decreased breadkdown of GLP C. It inhibits SGLT-2 and cause glucosuria D. It is amylin analogue and decrease glucagon
  • 43. 42. Least teratogenic antiepileptic drug in pregnancy is A. Valproate B. Phenytoin C. Carbamazepine D. Levetiracetam
  • 44. 43. A person presented to emergency in comatose state after consuming some unknown poison. Excessive sweating is apparent and on examination there is bradycardia and BP is 80/60 mmHg. Which of the following is antidote for treatment of this patient? a. Neostigmine b. Atropine c. Physostigmine d. N-acetylcysteine
  • 45. 44. Drug used for treatment of pregnancy induced hypertension is a. Alpha methyl dopa b. Lisinopril c. Hydralazine d. Telmisartan
  • 46. 45. A 56-year-old patient who is a known case of hypertension was being treated with antihypertensive drugs. He is having chronic cough since 2 weeks. He was started on antimicrobials but cough did not resolve. Which of the following antihypertensive drug may induce chronic cough in this patient? a. Lisinopril b. Minoxidil c. Propranolol d. Amlodipine
  • 47. 46. A patient presented with unilateral hemiparesis and presented to hospital after 8 hours. On examination and investigations, the blood pressure is 220/110 mmHg and HbA1c is 8%. CT scan of brain shows no hemorrhage. Which is the appropriate management of this patient? a. Conservative management b. tPA c. IV labetalol d. Sublingual nifedipine
  • 48. 47. A professor in medical college is teaching students about the adverse effects of a drug. Which of the following drug can cause gynaecomastia in men and menstrual irregularities in women? a. Spironolactone b. Propranolol c. Misoprostol d. Oxytocin`
  • 49. 48. A 25 year old male presented to emergency with head trauma due to road traffic accident. In hospital, the patient developed seizures. Emergency CT scan revealed widespread cerebral edema. Which of the following is diuretic of choice for cerebral edema in this patient? a. Mannitol b. Spironolactone c. Furosemide d. Hydrochlorthiazide
  • 50. 49. A 28 year old male went to Ladakh. He developed shortness of breath. Which of the following drug will you use for the treatment of mountain sickness at high altitude? a. Acetazolamide b. Furosemide c. Epleronone d. Salbutamol
  • 51. 50. A patient was operated for prostate carcinoma and during surgery local metastasis was noted. Which of the following drug this patient should receive? a. Leuprolide b. Desmopressin c. Octreotide d. Tamsulosin
  • 52. 51. A 30-year-old female presented with weight gain and cold intolerance. On investigations Value of T3 is 70 ng/dl (reference value 100-200 ng/dl),T4 is 3 mcg/dl (reference value 5-12 mcg/dl) and TSH is 20 mIU/L (reference value 0.5-5 mIU/L). Which of the following statement is correct regarding the management of this patient? a. Measure T3, T4 and TSH after one month b. Start L-thyroxine (T4) treatment 100 mcg once daily c. Start L-thyroxine (T4) treatment 25 mcg daily and gradually increase d. Start daily L-thyroxine (T4) 100 mcg with Liothyronine (T3) 5 mcg
  • 53. 52. A 50-year-old patient of type 2 diabetes mellitus was controlled on oral hypoglycemic drugs. This patient presented to hospital with profuse sweating and dizziness. There was presence of hypoglycemia. Which among the following drugs can result in hypoglycemia in this patient? a. Metformin b. Voglibose c. Vildagliptin d. Glipizide
  • 54. 53. Which of the following statements about the anti-psychotic drug, Aripiprazole is most appropriate? a. It is a D2 and 5HT 1A partial agonist with less extrapyramidal symptoms and less weight gain as compared to other anti-psychotic drugs b. It is a D2 and 5HT 1A partial agonist with more extrapyramidal symptoms and more weight gain as compared to other anti-psychotic drugs c. It is a D2 antagonist and 5HT2 agonist with less extrapyramidal symptoms and less weight gain as compared to other anti-psychotic drugs d. It is a D2 antagonist and 5HT2 agonist with less extrapyramidal symptoms and less weight gain as compared to other anti-psychotic drugs
  • 55. 54. A young boy was brought to hospital with complaints of few episodes of fast blinking and staring at something for around 20 seconds. ECG shows spike and wave pattern. Which of the following drug should be used for treatment? a. Phenytoin b. Sodium valproate c. Diazepam d. Carbamazepine
  • 56. 55. Which of the following options contains all intravenous hypnotic agents? a. Propofol, sodium thiopentone, Etomidate b. Halothane, sevoflurane, desflurane c. Suggamadex, Sufentanil, tramadol d. Vecuronium, Rocuronium, Atracurium
  • 57. 56. What is the mechanism of action of tPA? a. Fibrinolysis b. Inhibition of factor V and VII c. Inhibition of factor III and V d. Activation of anti-thrombin III
  • 58. 57. A patient with chronic obstructive pulmonary disease is on salmeterol therapy but there is no improvement. Which of the following drug should be added to the regimen? a. Inhaled Corticosteroids b. Tiotropium c. Oxygen therapy d. Promethazine
  • 59. 58. A patient is taking platinum-based chemotherapy for treatment of ovarian cancer. Which of the following drug should be used to prevent the nausea and vomiting in this patient? a. Domperidone b. Promethazine c. Palonosetron d. Metoclopramide
  • 60. 59. A patient has developed tuberculosis in which mycobacterium is resistant to rifampicin, isoniazid, kanamycin and levofloxacin. This tuberculosis is referred to as a. MDR TB b. Rifampicin resistant TB c. Extensively resistant TB d. Extremely resistant TB
  • 61. 60. A female patient presented with greenish vaginal discharge. Clue cells were present and Whiff test was positive in vaginal discharge. What is the drug of choice for this condition? a. Doxycycline b. Metronidazole c. Ceftriaxone d. Fluconazole
  • 62. 61. Which of the following drug will you omit from the prescription of a patient with XDR tuberculosis in pregnancy? a. Kanamycin b. Ethambutol c. Moxifloxacin d. Amoxycillin-clavulanic acid
  • 63. 62. A female patient was treated with clindamycin and developed diarrhea most probably pseudomembranous colitis. Which of the following is the likely organism responsible for this? a. Clostridium perfringens b. Clostridium difficile c. Clostridium tetani d. Clostridium septicum
  • 64. 63. Mechanism of action of doxycycline is a. DNA gyrase inhibitor b. Cell wall synthesis inhibitor c. Protein synthesis inhibitor d. DNA dependent RNA polymerase inhibitor
  • 65. 64. A 30-year-old man travelled to Assam. After coming back he developed fever, chills and rigors and altered sensorium. The peripheral smear demonstrated the presence of P. falciparum. What is the treatment of choice for this patient? a. Chloroquine, Sulfadoxine pyrimethamine b. Intravenous Artesunate c. Quinine d. Primaquine
  • 66. 65. A 50-year-old female presented to oncologist with breast lump. On examination and investigations, breast cancer was diagnosed. Echocardiography of the patient shows ejection fraction of 25 %. Which of the following drug will you avoid in this patient? a. Epirubicin b. Irinotecan c. Cyclophosphamide d. Methotrexate
  • 67. 66. Which of the following is an adverse effect of orlistat? a. Suicidal tendancy b. Steatorrhea c. Weight gain d. Hyperglycemia
  • 68. 67. What does LD50 signifies a. It is the dose at which 50% of animals will die b. It is the dose at which 50 animals will die c. It is the dose at which 50% patients will get the desired response d. It is drug that will be effective at 50% dose
  • 69. 68. All of the following are Side effects of Ritodrine except? a. Hypoglycemia b. Tachycardia c. Hypotension d. Pulmonary edema
  • 70. 69. A patient with history of snake bite presented in emergency with symptoms of fatigue and irritability. Ten vials of antivenom were administered intravenously but the patient is still having difficulty in swallowing, ptosis and respiratory distress. What should be the next line of management? a. Atropine and neostigmine b. Nebulizer with salbutamol c. IV hydrocortisone d. Antihistamines
  • 71. 70. A known patient of bronchial asthma was started on treatment with an anti-glaucoma drug. During the course of treatment, asthma worsened. Which drug is likely responsible for these symptoms? a. Timolol b. Brimonidine c. Latanoprost d. Pilocarpine
  • 72. 71. Which of the following is the fastest urate lowering therapy in tumor lysis syndrome to prevent renal damage? a. Allopurinol b. Febuxostat c. Rasburicase d. Furosemide
  • 73. 72. A patient presented with unstable angina and was started on 325 mg aspirin. What is correct about mechanism of action of this drug at this dose? a. It acts by inhibiting aggregation of platelets b. It acts by Inhibiting COX-1 in both endothelium and platelets c. It acts as a fibrinolytic agent d. It acts as an anti-coagulant agent
  • 74. 73. A known hypertensive patient came to emergency department with complaints of palpitations and shortness of breath. Her BP was 220/140. Among the option given below, which drug should not be used in treatment of this patient? a. I.V Nitro prusside b. I.V Esmolol c. I.V. Nicardipine d. I.V.Mannitol
  • 75. 74. A patient with chronic stable angina presents with pedal edema, breathlessness and other symptoms of heart failure. Which of the following drug can increase longevity in this patient? a. Nifedipine b. Digoxin c. Lisinopril d. Torsemide
  • 76. 75. A female washing clothes in cold water exhibited color change of her hands from pallor to red. Which of the following drug is best suited for this patient? a. ACE inhibitor b. Calcium channel blockers c. Thiazides d. Alpha blockers
  • 77. 76. What is the Drug of choice for Hyperthyroidism in first trimester of Pregnancy? a. Phosphorus iodine b. Methimazole c. Propylthiouracil d. Carbimazole
  • 78. 77. To minimize the risk of HPA axis suppression, What is the correct method of administration of prednisolone? a. Give at night just before bedtime b. Divide in three doses and give small doses 8 hourly c. Give on alternate day d. Replace with betamethasone
  • 79. 78. Which of the following drug used for pain relief in arthritis acts by both opioid and non opioid mechanisms? a. Paracetamol b. Tramadol c. Ketorolac d. Diclofenac
  • 80. 79. A patient with history of drug abuse presented to emergency in comatose state. Vitals recorded were; Respiratory rate 8/min, BP: 90/60 mmHg, Pulse 52 bpm. On examination, there was pin point pupil. What is the probable poisoning agent in this patient? a. Cocaine b. Morphine c. Alcohol d. Tricyclic antidepressant
  • 81. 80. A female visits the OPD to get rid of tobacco addiction Which of the following drug will you prescribe? a. Vareniciline b. Naloxone c. Disulfiram d. Flumazenil
  • 82. 81. A patient was started on lamotrigine 50 mg daily for treatment of epilepsy. Gradually, the dose of the drug is increased every 1-2 weeks to reach the final dose of 200 mg per day. This step up titration of lamotrigine dose is done in order to prevent or minimize the development of a. Sedation b. Sudden death c. Weight gain d. Skin rash
  • 83. 82. A 25-year-old female presented with mania. History revealed that her father is taking medications for bipolar disorder. Which of the following drug will you prescribe for this patient? a. Sodium valproate b. Lithium c. Carbamazepine d. Lamotrigine
  • 84. 83. A patient was administered a local anaesthetic agent and developed tachycardia, hypotension, arrhythmias and cardiovascular collapse. Which of the following should be used for treatment of this patient? a. 5% dextrose b. 20% dextrose c. 5% intralipid d. 20% intralipid
  • 85. 84. A patient presented to hospital with chest pain. ECG of the patient confirms it to be non ST elevation myocardial infarction. Which of the following drug useful in this patient acts by inhibiting platelet aggregation? a. Clopidogrel b. Streptokinase c. Low molecular weight heparin d. Warfarin
  • 86. 85. Which of the following anticoagulant is contraindicated in first trimester of pregnancy? a. Low molecular weight heparin b. Warfarin c. Unfractionated Heparin d. Enoxaparin
  • 87. 86. A 25-year-old male has frequent visits to rural areas and need to eat outside food. During one such visit, he developed loose stools. Which of the following drug should be used to manage this case? a. Metronidazole b. Diphenoxylate c. Loperamide d. Octreotide
  • 88. 87. Drug of choice for Legionnaire’s disease is: a. Azithromycin b. Tigecycline c. Streptomycin d. Amoxycillin
  • 89. 88. A patient developed COVID-19 pneumonia and was treated. Few months later, patient presented with nasal congestion and swelling of face. There were black lesions on the nose bridge. A diagnosis of mucormycosis was made. What is the drug of choice for this condition? a. Fluconazole b. Dexamethasone c. Amphotericin B d. Azithromycin
  • 90. 89. A female patient was taking warfarin for prosthetic heart valves. She became pregnant. What advise will you give regarding use of this drug in early pregnancy? A. Continue warfarin without any concern B. Continue warfarin but add vitamin K to the therapy C. Change warfarin to heparin D. Terminate the pregnancy
  • 91. 90. Which of the following drugs used in emergency management of asthma does not act by causing bronchodilaton? A. Salbutamol B. Ipratropium C. Deriphylline D. Hydrocortisone
  • 92. 91. Which of the following is the action of dopamine at low doses? A. It increases renal blood flow B. It causes vasoconstriction C. It has positive chronotropic action D. It causes hypotension
  • 93. 92. A female patient presented with pain and redness in great toe. Serum uric acid level is 9.6 mg/dL. Apart from prescribing analgesics for relieving pain, the physician prescribed a drug which can decrease the formation of uric acid. Which of the following enzyme is likely to be inhibited by this drug? A. Xanthine oxidase B. Thymidylate synthase C. Phosphpribosyl transferase D. DHFR
  • 94. 93. Vitamin that increases the absorption of iron from stomach is? A. Vitamin A B. Vitamin B12 C. Vitamin C D.Vitamin D
  • 95. 94. Longest acting phosphodiesterase inhibtor among the following drugs is? A. Sildenafil B. Vardenafil C. Tadalafil D. Phentolamine
  • 96. 95. Which of the following statements about these drugs is true? A. Drug A is most potent B. Drug A and B have equal efficacy but B is more potent than drug A C. Drug C is least potent D. Drug B has more potency but less efficacy than drug A
  • 97. 96. Which of the following anti-depressant drug acts by inhibiting the reuptake of both serotonin and nor-adrenaline? A. Fluoxetine B. Venlafaxine C. Phenelzine D. Nortriptyline
  • 98. 97. A 4-year-old boy drank full bottle of iron syrup. What is the antidote for acute iron poisoning? A. BAL B. EDTA C. Desferrioxamine D.Penicillamine
  • 99. 98. Which of the following antimicrobials is contraindicated in a patient with seizure disorder? A. Ampicillin B. Ofloxacin C. Doxycycline D.Cefixime
  • 100. 99. Which of the following drug is preferred for closure of ductus arteriosus in a preterm baby with PDA? A. Indomethacin B. Ibuprofen C. Mefenamic acid D.Paracetamol
  • 101. 100. Drug of choice for treatment of digoxin induced ventricular arrhythmias is? A. Atropine B. Lidocaine C. Amiodarone D.Procainamide
  • 102. 101. A patient was given halothane for general anaesthesia. He developed rigidity and hyperthermia. These symptoms are most likely due to which ion? A. Na+ B. K+ C. Ca2+ D.Cl–
  • 103. 102. A lady was stung by many bees. She developed breathing difficulty and hypotension. Drug used for treatment of this condition is? A. Adrenaline B. Nor-adrenaline C. Dopamine D.Isoprenaline
  • 104. 103. A patient presented to hospital with road traffic accident. The patient has raised ICP without intracranial hematoma. Which of the following is the preferred drug for the management of this patient? A. Mannitol B. Prednisolone C. Glycerol D.Furosemide
  • 105. 104. A patient of COVID-19 requires oxygen therapy. Which of the following cylinders should be used to provide oxygen to this patient? A B C D
  • 106. 105. A drug addict patient presented with agitation and ulceration on skin. The patient had severe itching and scratch marks on the body. On examination there is presence of tachycardia, mydriasis and hypertension. Likely substance responsible for these symptoms is? A. Cocaine B. Cannabis C. LSD D.Heroin
  • 107. 106. A patient on treatment of myasthenia gravis presents with increasing muscle weakness. Which of the following drug is used to differentiate myasthenia gravis from cholinergic crisis? A. Edrophonium B. Pilocarpine C. Atropine D.Pyridostigmine
  • 108. 107. A patient had a history of previous child with down syndrome, currently on lithium therapy present for antenatal checkup. Which of the following condition is the fetus at the greatest risk of? A. Ebstein’s anomaly B. Down’s syndrome C. Tetralogy of Fallot D.Neural tube defect
  • 109. 108. Hormones and endogenous substances that are involved in regulation of calcium balance in the body are? A. Vitamin D, PTH and insulin B. Vitamin D, PTH and calcitonin C. Vitamin D, calcitonin and TSH D. Vitamin D, PTH and TSH
  • 110. 109. Which of the following drugs can be used for the treatment of undescended testes? A. Testosterone B. GnRH C. Cold water compress D.Anti-MIH therapy
  • 111. 110. After eating an unknown plant, A child presented to emergency in comatose state. The pupils were dilated and skin was dry. Body temperature is raised and heart rate is high. These signs and symptoms can occur with the intake of which of the following substance? A. Dhatura B. Poppy seeds C. Mushroom D.Cannabis
  • 112. 111. A farmer presented to emergency with garlic odour, pin point pupil and excessive secretions. Which of the following is drug of choice for treatment of this poisoning? A. Atropine B. Flumazenil C. N-acetylcysteine D.Oxime
  • 113. 112. An elderly male developed the symptoms of dementia and was diagnosed as Alzhiemer’s disease. Which neurotransmitter level is reduced in this disease? A. Dopamine B. Acetylcholine C. Adrenaline D.Serotonin
  • 114. 113. An elderly patient presented with tremors, rigidity and bradykinesia. The most likely neurotransmitter affected in the brain of this patient is? A. Dopamine B. Acetylcholine C. Adrenaline D.Serotonin
  • 115. 114. Which of the following combination of seizure and its first line drug is not correct? A. Generalised seizure - Valproate B. Myoclonic - Topiramate C. Focal - Levetiracetam D.Absence - Ethosuximide
  • 116. 115. Aspirin may increase serum uric acid. What is the mechanism for this action? A. It inhibits reabsorption of uric acid from renal tubules B. It results in increased production of uric acid C. It interferes with filtration of uric acid through glomerulus D. It causes breakdown of proteins leading to excessive uric acid production
  • 117. 116. Which of the following drug is given to mother delivering premature baby for fetal lung maturation? A. Aspirin B. Dexamethasone C. Magnesium sulfate D. Depot medroxy progesterone acetate
  • 118. 117. A patient presented with hospital acquired pneumonia. On testing he was found to be allergic to penicillin G. Which of the following drug is likely to be safe in this patient? A. Ceftriaxone B. Ampicillin C. Aztreonam D. Imipenem
  • 119. 118. A patient was taking theophylline for bronchial asthma. After starting another drug, symptoms of theophylline toxicity started appearing. Which is the likely drug started? A. Erythromycin B. Phenytoin C. Griseofulvin D. Rifampicin
  • 120. 119. Which intravenous anaesthetic is used for day care surgery? A. Ketamine B. Propofol C. Thiopentone D. Etomdiate
  • 121. 120. Which of the following drug is associated with post operative delirium and hallucinations? A. Ketamine B. B. Thiopentone C. C. Fentanyl D. D. Halothane
  • 122. 121. A 65 years old patient presented with dribbling of urine with urgency. He was started on prazosin therapy but the patient developed postural hypotension. Which of the following is the better alternative drug for this patient? A. Terazosin B. Tamsulosin C. Timolol D. Phenoxybenzamine
  • 123. 122. Which of the following is a selective estrogen receptor modulator? A. Tamoxifen B. Anastrozole C. Mifepristone D. Ethinyl estradiol
  • 124. 123. Which among the given options is a monoclonal antibody used for treatment of cancer? A. Cisplatin B. Rituximab C. Vincristine D. Cyclophosphamide
  • 125. 124. A new drug is introduced in the market after which phase of clinical trials A. Phase 1 B. Phase 2 C. Phase 3 D. Phase 4
  • 126. 125. Identify the missing substance X marked in the box in the given diagram. A. G protein B. Protein C C. Cyclic AMP D. Calcium
  • 127. 126. In the given diagram, identify drug Z marked by the arrow A. Clopidogrel B. Heparin C. Dicumarol D. Alteplase
  • 128. 127. Excessive phytates intake in the diet may lead to deficiency of A. Proteins B. Iron C. Vitamin B12 D. Vitamin D
  • 129. 128. Beta blockers are avoided in all of the following conditions except A. Glaucoma B. Peripheral vascular disease C. Diabetes mellitus D. COPD
  • 130. 129. A 22 year old female presents 8 hours after sexual assault. It is the 13th day of her menstrual cycle. Which emergency contraceptive should be prescribed to her? A. Levonorgestrel 1.5 g single tablet B. Injection DMPA C. OCP from day 1 of next cycle D. Misoprostol
  • 131. 130. A patient presented with the following features after chronic intake of a drug for several years. Which is the likely drug person has consumed? A. Paracetamol B. Prednisolone C. Phenytoin D. Metformin
  • 132. 131. Which of the following is wrongly matched? a.Dabigatran: Idarucizumab b.Rivaroxaban: Andexanet alpha c.Fondaparinux: Ciraparantag d.Apixaban: Andexanet alpha
  • 133. 132. A patient had symptoms of redness and photophobia. On examination, cells were present in anterior chamber with keratic precipitates. Intraocular pressure was measured to be 38 mm Hg. Which of the following anti-glaucoma drug should be avoided? a.PG analogues b.Beta blockers c.Mannitol d.Carbonic anhydrase inhibitors
  • 134. 133. A hypertensive patient has grade 4 renal failure and GFR less than 30 mL/min. The physician wants to prescribe a thiazide diuretic. Which is the best drug for this patient? a.Hydrochlorthiazide b.Chlorthalidone c.Metolazone d.Indapamide
  • 135. 134. A study was conducted to see the effect of different drugs on isolated mammalian intestinal tissue in Dale’s organ bath. The following graph is obtained. Which of the following is the likely drug? a.Acetylcholine b.Barium chloride c.Adrenaline d.KCl
  • 136. 135. Nivolumab is a monoclonal antibody used for the treatment of a.Hodgkin’s lymphoma b.Medulloblastoma c.Retinoblastoma d.Pleuropulmonary blastoma
  • 137. 136. A patient was taking chemotherapy cyclophosphamide, methotrexate and 5-Fluorouracil for treatment of breast cancer. She developed fever and on investigations, She had anemia and neutropenia. Which of the following antimicrobial is not indicated in this patient for treatment of this infection? a.Piperacillin-tazobactam b.Cefepime c.Linezolide d.Meropenem
  • 138. 137. A patient on retigabine therapy for a month for focal seizures. Phenytoin was added to therapy. What is the next step? a.Change retigabine to carbamazepine b.Decrease dose c.Increase dose d.Stop retigabine
  • 139. 138. Which of the following drugs can be used for the treatment of severe covid-19 pneumonia in children? a.Steroids b.Remdesivir c.Ivermectin d.All of these
  • 140. 139. We administer a drug following first order kinetics. If the administered dose is doubled? a.Plasma concentration and elimination half life remains same b.Plasma concentration becomes double and elimination half life remains same c.Elimination half life becomes double and plasma concentration remains same d.Elimination half life and plasma concentration both becomes double
  • 141. 140. Identify the mechanism of action of vancomycin from the given figure:
  • 142. 141. For treatment of which of the following conditions, a combination of antimicrobials is usually not required? a.Malaria b.Gonorrhea c.Intra-abdominal infection d.Tuberculosis
  • 143. 142. Topiramate is used for the treatment of a.Lennox Gastaut syndrome b.Attention deficit hyperkinetic disorder c.Treatment of migraine d.Prophylaxis of heat stroke
  • 144. 143. Which of the following is FDA-approved drug for long term treatment of obesity? a.Sibutramine b.Liraglutide c.Metformin d.Fenfluramine
  • 145. 144. A newly diagnosed tuberculosis patients was found to be HIV positive. Which of the following statement is true about treatment of this patient? a. Start ATT and ART together b. ART should be started only if CD4 count is less than 50/mcL c. ATT should be started first followed by ART 2 weeks later d. ART should be started first followed by ATT 2 weeks later
  • 146. 145. A patient with stroke was started on clopidogrel. He developed another episode of stroke 6 months later. Which of the following is the likely cause? a.Rapid metabolism by CYP 1A2 b.Poor metabolism by CYP 2C19 c.Poor metabolism by CYP 2D6 d.Poor metabolism by CYP 2E1
  • 147. 146. Which of the following is true about nicotine replacement therapy? a. Vareniciline is a form of NRT that comes with black box warning of cardiovascular adverse effects b. There should be a gap of minimum 15 minutes between nicotine and coffee or acidic food c. NRT is given by GI route d. Nicotine gum is better because it attains 10-15 percent higher plasma concentration than lozenges
  • 148. 147. A patient was given ipratropium and he developed paradoxical bronchoconstriction. Which is not a likely mechanism of this bronchoconstriction? a.Benzalkonium chloride b.Presynaptic stimulation of M2 autoreceptors? c.Use of hypertonic saline in nebuliser d.EDTA
  • 149. 148. Mechanism of action of local anaesthetics is a.Alters the resting membrane potential of the cell b.Decrease transport of K across cell c.Decrease transport of Na via voltage gated channels d.Binds to receptor complex and prevents activation of gates
  • 150. 149. Which of the following factor in PC vs time graph is a measure of extent of drug absorption of a drug? a. Area under the curve b. Half life c. Cmax d. Tmax
  • 151. 150. Manufacturer of a drug company labels the drug contains 500 mg paracetamol. On quantitative analysis by the authorities, it was found to contain only 200 mg of drug. According to drugs and cosmetics act 1940, this type of drug is known as? a. Spurious drug b. Adulterant drug c. Unethical drug d. Misbranded drug
  • 152. 151. Which of the following statement is correct regarding the given graph? a. Drug A represents agonist and Drug B represents inverse agonist b. Drug C represents agonist and Drug D is inverse agonist c. Drug A is agonist and Drug D is inverse agonist d. Drug B is partial agonist and Drug C is inverse agonist
  • 153. 152. Type of stimulatory G protein in PIP2-Phospholipase activation pathway is a. Gs b. Gi c. Gq d. Go
  • 154. 153. Identify the type of inhibition from the given graph a. Competitive b. Non-competitive c. Un-competitive d. Allosteric
  • 155. 154. A 20-year-old boy was brought to emergency room after consuming some unknown substance. The patient was sweating profusely, saliva was drooling from mouth and tears were present. The boy had involuntary urination and diarrhea. On examination, the heart rate was 58 beats per minute and blood pressure was 80/60 mm Hg. Which of the following is the drug of choice for treatment of this patient? a. Atropine b. N-acetylcysteine c. Naloxone d. Physostigmine
  • 156. 155. A patient with pre-existing liver disease consumed a drug and developed worsening of liver dysfunction. The metabolite of the drug responsible is N-acetyl-para-amino-benzo-quinone-imine. The likely implicated drug in this case is a. Paracetamol b. Valproate c. Amiodarone d. Lorazepam
  • 157. 156. Identify the correct match regarding the drug and its adverse drug reaction? 1. Hydralazine: Heart failure 2 Verapamil: Constipation 3. Aliskiren: Hypokalemia 4. Atenolol: Hemolytic anemia
  • 158. 157. Which of the following anti-diabetic drugs is associated with increased risk of fractures in a female with osteoporosis? a. Canagliflozin b. Rosiglitazone c. Voglibose d. Rapaglinide
  • 159. 158. Half life of letrozole is a. 45 hours b. 72 hours c. 96 hours d. 120 hours
  • 160. 159. Which of the following statement/statements regarding use of 5-alpha reductase inhibitors in BPH are correct? (multiple correct) a. Decrease in serum PSA b. Increase in serum PSA c. Decrease in cellular testosterone d. Decrease in cellular DHT
  • 161. 160. A 42 year old chronic alcoholic presents to emergency department with altered sensorium and seizures. He has not consumed alcohol for last two days. On investigations, his serum ALT is 150, AST is 180 and GGT is 563 units per litre. Best drug for first line management of this patient is? a. Lorazepam b. Diazepam c. Clonazepam d. Alprazolam
  • 162. 161. ACTH is the treatment of a. Juvenile myoclonic epilepsy b. West syndrome c. Dravet syndrome d. Lennox Gastaut syndrome
  • 163. 162. A patient of psychosis was being treated with risperidone. He presented to emergency with upward fixed gaze. What will be the treatment of this patient? a. Wait and assurance b. Intramuscular Promethazine c. Injection Diazepam d. Injection Lorazepam
  • 164. 163. A 30 year old female presented to psychiatry OPD with symptoms of hypomania. She has a past history of mania and wants to conceive. Which of the following drug is most teratogenic? a. Valproate b. Lithium c. Carbamazepine d. Olanzapine
  • 165. 164. Which of the following options correctly represent the increasing order of potency of inhalational anaesthetic agents? a. N2O < Isoflurane < Halothane < Methoxyflurane b. Methoxyflurane < Halothane < Isoflurane < N2O c. Halothane < Isoflurane < Methoxyflurane < N2O d. Isoflurane < N2O < Halothane < Methoxyflurane
  • 166. 165. From the given diagram, identify the mechanism of action of isoniazid? a. Drug A b. Drug B c. Drug C d. Drug D
  • 167. 166. The given diagram shows the steps in formation of bacterial cell wall. Identify the site of action of beta lactam antimicrobials. a. A b. B c. C d. D
  • 168. 167. Pregnant female with history of lepromatous leprosy presents to OPD with type 2 lepra reaction. What should be the management? a. Thalidomide b. Stop MDT and start steroids c. Add corticosteroids d. Antibiotics
  • 169. 168. Active metabolite of cyclophosphamide is a. 4-ketoacyl cyclophosphamide b. 4-hydroxy cyclophosphamide c. N-Acetyl cyclophosphamide d. N-Methylcyclophosphamide
  • 170. 169. Immune checkpoint inhibitor approved for treatment of advanced endometrial carcinoma is? a. Pembrolizumab b. Nivolumab c. Ipilimumab d. Trastuzumab
  • 171. 170. Which of the following is not an immune check point inhibitor? a. Cetuximab b. Pembrolizumab c. Atezolizumab d. Nivolumab
  • 172. 171. Which of the following drug is not indicated in treatment of Sickle cell anemia? a. Bebtelovimab b. L-glutamine c. Hydroxyurea d. Voxeletor
  • 173. 172. Elimination rate constant of a drug is 0.05/hr. What is its half life? A. 6.5 hr B. 20 hr C. 13.9 hr D. 8 hr
  • 174. 173. Anionic and slightly acidic drugs usually bind to A. Albumin B. Alpha acid glycoprotein C. Ceruloplasmin D. Globulin
  • 175. 174. The following table gives the data of AUC of drug A alone and AUC of drug A when combined with drug B. p value is <0.01. Which of the following statement regarding these drugs is most correct? A. Drug B decreases the first pass metabolism of drug A B. Drug B increases the systemic metabolism of drug A C. Drug B decreases the intestinal absorption of drug A D. Drug B increases the renal clearance of drug A Drug Area Under the Curve (AUC) A alone 550 ± 150 A plus B 850 ± 150
  • 176. 175. Category A, B, C, D, X division of drugs is based on A. Safety in pregnancy B. Dose adjustment in renal failure C. Therapeutic index and safety D. Over the counter use of drug
  • 177. 176. Which dose of dopamine act preferably on beta-1 receptors? A. Less than 2 mcg B. 2-10 mcg C. 10-20 mcg D. More than 20 mcg
  • 178. 177. A patient presents with grade 2 pulmonary artery hypertension. Vaso-reactive stimulation test is negative. Which is the most preferred initial management of this patient? A. Epoprostenol B. Amlodipine C. Alprostadil D. Ambrisentan
  • 179. 178. Resistant hypertension is defined as inability to attain goal blood pressure inspite of the concurrent use of 3 antihypertensive agents of different classes prescribed at optimal doses including A. Alpha blockers B. Diuretics C. Reserpine D. Alpha Methyl Dopa
  • 180. 179. A 65-year-old man comes to OPD with history of fall. He is hypertensive with history of atrial fibrillation and is presently on captopril, atenolol, aspirin and amiodarone. He presents with the following finding as shown in the image. What is the most probable diagnosis? A. Silver toxicity B. Lupus pernio C. Amiodarone induced skin lesion D. Captopril toxicity
  • 181. 180. A patient of chronic liver cirrhosis presents with ascites. Which of the following is the best diuretic to be used in this patient? A. Eplerenone B. Chlorthiazide C. Furosemide D. Triamterene
  • 182. 181. What is the prenatal dose of dexamethasone given for lung maturation in premature infants ? A. 6 mg 2 doses 12 hours apart B. 12 mg 2 doses 24 hours apart C. 6 mg 4 doses 12 hours apart D. 6 mg 4 doses 24 hours apart
  • 183. 182. A 60 year old female presents to hospital for routine check-up. She attained menopause at 52 years of age and has a past history of Colle’s fracture. DEXA scan was done and T-score is -2.5. What should be the treatment given to this patient? A. Vitamin D and Calcium supplementation B. Alendronate C. Repeat DEXA Scan D. Hormone replacement therapy
  • 184. 183. Dose of carbetocin for post partum hemorrhage is A. 50 mcg iv over 2 min B. 100 mcg iv over 1 min C. 150 mcg im over 2 min D. 200 mcg iv over 1 min
  • 185. 184. Which of the following drug is the only medication useful in preventing disability progression in a patient with primary progressive multiple sclerosis? A. Natalizumab B. Ocrelizumab C. Siponimod D. Rituximab
  • 186. 185. Most effective cessation agent when used as monotherapy for smoking cessation is? A. Varenicline B. Nicotine gum C. Sustained release Bupropion D. Nicotine Patch
  • 187. 186. Which of the following is a novel antidepressant drug? A. Vilazodone B. Asenapine C. Flibanserin D. Lurasidone
  • 188. 187. Which of the following antidepressant drug should be avoided in a patient of angle closure glaucoma? A. Sertraline B. Amitriptyline C. Mirtazapine D. Venlafaxine
  • 189. 188. Which of the following anti-epileptic drug is used in treatment of neuropathic pain ? A. Gabapentin B. Lamotrigine C. Carbamazepine D. Pregabalin a) A,C,D b) A,B,C c) A,B d) A,C
  • 190. 189. Which of the following drugs improves bioavailability and prolongs duration of action of Saquinavir? A. Ritonavir B. Cimetidine C. Vitamin C D. Remdesivir
  • 191. 190. A patient on phenytoin for seizure disorder was prescribed sucralfate 4 times a day for peptic ulcer. What should be the minimum duration between consumption of two drugs? A. 30 min B. 60 min C. 90 min D. 120 min
  • 192. 191. Mechanism of action of Remdesivir is inhibition of? A. RNA dependent RNA polymerase B. DNA dependent RNA polymerase C. Viral protease enzyme D. Cell wall synthesis
  • 193. 192. Which of the following cephalosporins can increase the effect of warfarin resulting in raised INR and increased risk of bleeding? A. Cefoperazone B. Cefixime C. Ceftibiprole D. Ceftazidime
  • 194. 193. A female patient presented with greenish vaginal discharge and pruritus. On colposcopy, strawberry cervix is noted. What is the drug of choice for this condition? A. Ceftriaxone B. Metronidazole C. Acyclovir D. Fluconazole
  • 195. 194. What is mechanism of action of Cyclosporine? A. Calcineurin inhibitor B. mTOR inhibitor C. IL-2 receptor antagonist D. TNF-α inhibitor
  • 196. 195. A patient was given intravenous botulinum toxin and the patient died. Mechanism of botulinum toxin overdose is A. Inhibits release of acetylcholine B. Inhibit reuptake of nor-epinephrine C. Blockade of post synaptic nicotinic cholinergic receptors D. Inhibit entry of acetylcholine in vesicle
  • 197. 196. Drug of choice in paracetamol overdose is a. N-acetylcysteine b. Dopamine c. Hydralazine d. Furosemide
  • 198. 197. A young female presented with left sided severe throbbing headache associated with nausea, vomiting, photophobia and phonophobia. Which of the following drug can provide immediate relief to this patient? a. Propranolol b. Sumatriptan c. Topiramate d. Flunarizine
  • 199. 198. A patient on digoxin therapy accidently consumed 8 tablets of digoxin 0.25 mg. Two hours later, he presented to emergency with heart rate of 54 bpm and ECG evidence of third-degree AV block. What is the immediate management of this patient? a. Digoxin immune Fab b. Lignocaine c. Phenytoin d. DC cardioversion
  • 200. 199. Which of the following hypolipidemic drug acts by inhibition of PCSK-9? a. Atorvastatin b. Evolocumab c. Ezetimibe d. Lomitapide
  • 201. 200. A patient presents with pituitary tumor that overproduces growth hormone. surgical removal of the tumor was incomplete. What is the first line treatment of this patient? a. Leuprolide b. Octreotide c. Nafarelin d. Goserelin
  • 202. 201. A 40-year-old male presents with protrusion of chin, excessive sweating, impaired glucose tolerance and enlargement of hands and feet? Which of the following drugs is a growth hormone receptor antagonist used to treat this condition? a. Pegvisomant b. Octreotide c. Cabergoline d. Olcegepant
  • 203. 202. A 40-year-old diabetic female presented to emergency with abdominal pain, vomiting and recent onset confusion. On examination, she had irregular breathing and dehydration. Her blood sugar is 539 mg/dl and there was presence of ketone bodies in the urine. Blood pressure of the patient is 80/50 mm Hg. What is the next best step in the management of this patient? a. Regular insulin b. Intravenous fluids c. Intravenous fluids with regular insulin d. Long acting insulin
  • 204. 203. A female presented with galactorrhea. Her urine pregnancy test was negative. MRI of head revealed a large pituitary tumor. Patient refused to undergo surgery for the tumor. Which of the following is the best drug for the treatment of this patient? a. Octreotide b. Bromocriptine c. Promethazine d. Clozapine
  • 205. 204. A patient with opioid poisoning presents with severe respiratory depression. What is the most effective drug for treatment of this patient? a. Fomepizole b. Naltrexone c. Flumazenil d. Naloxone
  • 206. 205. A patient on anti-depressant therapy presents with elevated body temperature, dilated pupil, palpitations and low blood pressure. ECG shows tachycardia, broad QRS complex and right axis deviation. Which of the following interventions must be done immediately? a. Wait and watch b. Intravenous sodium bicarbonate c. Intravenous esmolol d. DC cardioversion
  • 207. 206. A patient presents with tremors, rigidity and bradykinesia. Which of the following drugs can be used for the treatment of this patient? a. Selegiline b. Donepezil c. Fluoxetine d. Haloperidol
  • 208. 207. A child born by normal vaginal delivery developed repeated attacks of flexion of neck over the trunk and jerks in the hands. EEG shows the presence of hypsarrhythmia. Anti-epileptic drug of choice for this patient is? a. Phenobarbitone b. Phenytoin c. ACTH d. Levetiracetam
  • 209. 208. An 11-year-old boy presented to emergency with vomiting. Parents gave history of consuming 10-15 tablets of ferrous sulphate a day before. What is the antidote of iron for treatment of this patient? a. Dimercaprol b. Desferrioxamine c. d- Penicillamine d. Activated charcoal
  • 210. 209. A deep vein thrombosis patient was started on an anticoagulant therapy. Next day, the patient presented with the features shown in the diagram below. Likely drug implicated for this adverse effect is a. Heparin b. Rivaroxaban c. Warfarin d. Dabigatran
  • 211. 210. A patient with COPD was given intravenous aminophylline therapy following which the patient developed diuresis. This is due to action on which effector? a. Adenosine receptors b. Beta 2 adrenergic receptors c. IL-10 d. Histone deacetylase
  • 212. 211. A female with ovarian cancer was on cisplatin therapy. He presented to hospital with several episodes of vomiting. After giving an antiemetic drug, the patient developed dystonia. The anti-emetic drug most likely to be responsible for these features is a. Metoclopramide b. Meclizine c. Ondansetron d. Scopolamine
  • 213. 212. A pregnant female with urinary tract infection took antimicrobials for the same. The baby of this female developed tendon rupture and arthropathy. What is the likely mechanism of action of the antimicrobial consumed by the pregnant female? a.Folic acid synthesis inhibitor b.Mycolic acid synthesis inhibitor c. DNA gyrase inhibitor d.DNA inhibitor
  • 214. 213. A 48 year old patient presented to OPD with tingling and numbness of fingers. The patient is chronic alcoholic and was recently started on antitubercular treatment. What should be the treatment of this patient? a. Vitamin B3 b. Vitamin B6 c. Vitamin B1 d. Vitamin B12
  • 215. 214. In MDR tuberculosis, which of following drug combinations show resistance? a. Rifampicin and Fluoroquinolones b. Isoniazid and rifampicin c. Isoniazid and Pyrazinamide d. Rifampicin and kanamycin
  • 216. 215. A patient presented with hyperkalemia. Which of the following drugs can cause intracellular movement of and thus decrease in extracellular potassium levels? a. Atropine b. Lactic acid c. Epinephrine d. Glucagon
  • 217. 216. Oral Factor Xa Inhibitor is: a. Dabigatran etexilate b. Rivaroxaban c. Fondaparinux d. Bivalirudin
  • 218. 217. High plasma protein binding of a drug results in: a. Decreased glomerular filtration b. High volume of distribution c. Lowers duration of action d. Less drug interaction
  • 219. 218. DOC for prophylaxis against Diphtheria: a. Erythromycin b. Rifampicin c. Cloxacillin d. Ciprofloxacin
  • 220. 219. Variation in drug responsiveness to a particular dose of the drug in different individuals can be obtained from: a. Graded Dose Response Curve b. Efficacy c. Potency d. Quantal Dose Response Curve
  • 221. 220. Most specific antiemetic for Chemotherapy induced vomiting is: a. Doxylamine b. Tegaserod c. Granisetron d. Domperidone
  • 222. 221. Identify the true statement regarding Clinical Trials: a. Phase 1 is done to determine efficacy in patients b. Healthy volunteers are recruited for the first time in Phase II c. Randomized Controlled Trials in patients is done in Phase III d. Phase IV is Pharmacokinetics study in animals
  • 223. 222. Fluoroquinolone with highest oral bioavailability: a. Levofloxacin b. Gemifloxacin c. Ciprofloxacin d. Norfloxacin
  • 224. 223. Which of the following statement is true regarding Fixed Dose Combination of drugs? A. Adverse effect of one drug may be reduced by the other drug B. Two drugs with different pharmacokinetics can be combined easily C. Dose of one drug can be altered independently as per requirement D. Adverse effect can be ascribed to a single drug
  • 225. 224. Which of the following is a K+ Channel Opener? A. Ranolazine B. Nicorandil C. Verapamil D. Quinidine
  • 226. 225. Tolvaptan is used for: A. SIADH B. Central DI C. Von Willebrand Disease D. Catecholamine resistant Shock
  • 227. 226. Mechanism of action of Oseltamivir: A. DNA polymerase inhibition B. Inhibition of viral mRNA C. Blocking viral uncoating D. Neuraminidase inhibition
  • 228. 227. Buspirone acts on: A. 5HT1A B. 5HT1B C. 5HT2 D. 5HT3
  • 229. 228. Carbapenem with maximum seizure risk is: A. Imipenem B. Meropenem C. Ertapenem D. Doripenem
  • 230. 229. Cilastatin is given in combination with Imipenem because: A. Cilastatin prevents degradation of Imipenem in kidney B. Cilastatin increases absorption of Imipenem C. Inhibits the enzymes that digest Imipenem in stomach D. Reduces side effects of Imipenem.
  • 231. 230. Mechanism of action of Triazoles: A. Inhibits ergosterol biosynthesis B. Inhibits tubulin C. Inhibits glucan synthesis D. Inhibits cell wall synthesis
  • 232. 231. Identify the false statement regarding Teduglutide is: A. Used for Short-Bowel Syndrome B. GLP2 agonist C. A pancreatic enzyme D. A gut Hormone
  • 233. 232. Hypertension and pulmonary edema associated with scorpion sting is managed by: A. Carvedilol B. Prazosin C. Spironolactone D. Phentolamine
  • 234. 233. Mannitol is used in the management of: A. Congestive Cardiac Failure B. Acute Congestive Glaucoma C. Acute Renal Failure D. Pulmonary edema
  • 235. 234. Drug inhibiting granulocyte migration is: A. Montelukast B. Cromoglycate C. Colchicine D. Felbamate
  • 236. 235. Drug capable of causing ocular hypotension with apnoea in infants is: A. Acetazolamide B. Latanoprost C. Brimonidine D. Apraclonidine
  • 237. 236. Inhalational anesthetic with highest respiratory irritation is: A. Desflurane B. Nitrous Oxide C. Sevoflurane D. Halothane
  • 238. 237. Drug used for smoking cessation: A. Varenicline B. Acamprosate C. Nalmefene D. Gabapentin
  • 239. 238. Indication for use of Pegylated Filgrastim is in the correction of: A. Anaemia B. Neutropenia C. Thrombocytopenia D. Pancytopenia
  • 240. 239. Agent used for eliciting diagnostic differentiation of Myasthenia Gravis from Cholinergic crisis is: A. Ecothiophate B. Edrophonium C. Neostigmine D. Ambenonium
  • 241. 240. Anesthetic not painful on Intravenous administration is: A. Ketamine B. Propofol C. Etomidate D. Methohexital
  • 242. 241. Which IV anesthetic does not cause cardiac depression? A. Etomidate B. Ketamine C. Methohexital D. Thiopentone
  • 243. 242. Mechanism of action of curare like drugs: A. Competitive, Non depolarizing block at the Nm cholinergic receptors B. Noncompetitive, Non depolarizing block at the Nm cholinergic receptors C. Competitive, Depolarizing block at the Nm cholinergic receptors D. Noncompetitive, Depolarizing block at the Nm cholinergic receptors
  • 244. 243. Reason for preferring cis-Atracurium over Atracurium is: A. Equal potency like Atracurium B. Lesser provocation of histamine release C. Short fast action D. Does not undergo Hoffman elimination
  • 245. 244. Drug of Choice for Digoxin induced Ventricular Tachycardia: A. Propranolol B. Diltiazem C. Verapamil D. Lignocaine
  • 246. 245. Contraindications to use of Beta Blockers: A. Glaucoma B. Tachycardia C. Bronchial asthma D. Hypertension
  • 247. 246. Drug acting via tyrosine kinase receptor is A. TRH B. TSH C. Insulin D. MSH
  • 248. 247. A patient on lithium therapy developed hypertension. He was started on thiazides for hypertension. After few days, he developed coarse tremors and other symptoms suggestive of lithium toxicity. Explain the likely mechanism of this interaction. A. Thiazide inhibits the metabolism of lithium B. Thiazides act as an add on drug to lithium C. Thiazides increase the tubular reabsorption of lithium D. Thiazides cause loss of water thereby increased lithium levels.
  • 249. 248. Which of the following drugs shown in the graph below has highest potency? A. Drug A B. Drug B C. Drug C D. Both Drug A and B
  • 250. 249. Mechanism of resistance to penicillins via beta lactamase is A. Altered penicillin binding proteins B. Drug efflux C. Breaks drug structure D. Alteration in 50S ribosome structure
  • 251. 250. Which of the following drugs act by inhibiting DNA replication? A. 6 Mercaptopurine B. Actinomycin D C. Mitomycin C D. Asparaginase
  • 252. 251. A female patient was on lithium therapy for bipolar disorder for 6 months. She kept the fast for few days due to religious reasons and presented with seizures, coarse tremors, confusion and weakness of limbs. Which of the following should be done to diagnose her condition? A. Serum Electrolytes B. Serum lithium levels C. ECG D. MRI
  • 253. 252. Which of the following drugs is used as nail lacquer for fungal infections A. Fluconazole B. Nystatin C. Itraconazole D. Terbinafine
  • 254. 253. Pegloticase is used for treatment of :- A. Ankylosing spondylosis B. CPPD C. Chronic tophaceous gout D. Refractory Rheumatoid arthritis
  • 255. 254. Which of the following is not a prokinetic? A. 5HT4 agonist B. D2 blocker C. Macrolides D. Diphenoxymethane
  • 256. 255. Fluoroquinolone contraindicated in liver disease is A. Levofloxain B. Pefloxacin C. Ofloxacin D. Lomefloxacin
  • 257. 256. At a high altitude of 3000m, a person complains of breathlessness. All of the following can be used for management of this person except A. Intravenous digoxin B. Oxygen supplementation C. Immediate descent D. Acetazolamide
  • 258. 257. True about non-competitive inhibition is A. Km remains same, Vmax decreases B. Km increases, Vmax remains same C. Km decreases, Vmax increases D. Km increases, Vmax increases
  • 259. 258. Theophylline causes diuresis because of A. PDE inhibition B. Adenosine A1 receptor antagonism C. Beta 2 agonism D. M3 receptor antagonism
  • 260. 259. A patient with diabetes developed post-operative urinary retention. Which of the following drugs can be used for short term treatment to relieve the symptoms of this person? A. Bethanechol B. Methacholine C. Terazosin D. Tamsulosin
  • 261. 260. Drug of choice for invasive aspergillosis is A. Posaconazole B. Voriconazole C. Liposomal AMB D. Caspofungin
  • 262. 261. Which of the following antihypertensive drug is avoided in patients with high serum uric acid levels? A. Hydrochlorothiazide B. Enalapril C. Prazosin D. Atenolol
  • 263. 262. Which of the following drug acts by inhibiting the transcription of DNA to RNA? A. Rifampicin B. Nitrofurantoin C. Ciprofloxacin D. Novobiocin
  • 264. 263. A patient was recently started on Fluphenazine. Few weeks later, he developed tremors, rigidity, bradykinesia and excessive salivation. First line of management for this patient is A. Selegiline B. Trihexyphenidyl C. Pramipexole D. Amantadine
  • 265. 264. Which of the following antimicrobials should not be given to a chronic asthmatic patient managed on theophylline therapy? A. Erythromycin B. Cefotaxime C. Cotrimoxazole D. Amoxicillin
  • 266. 265. A person was given a muscle relaxant that competitively blocks nicotinic receptors. Which of the following drug is used for reversal of muscle relaxation after surgery? A. Neostigmine B. Carbachol C. Succinylcholine D. Physostigmine
  • 267. 266. Anti-glaucoma drug that acts by increasing uveoscleral outlfow is A. Latanoprost B. Timolol C. Pilocarpine D. Dorzolamide
  • 268. 267. A patient diagnosed with Rheumatoid arthritis was on medications. After 2 years, he developed blurring of vision and was found to have corneal opacity. Which drug is most likely to cause this? A. Sulfasalazine B. Chloroquine C. Methotrexate D. Leflunomide
  • 269. 268. A boy is planning to travel by bus. Which of the following drug can be used to prevent motion sickness in this person? A. Promethazine B. Cetrizine C. Loratidine D. Fexofenadine
  • 270. 269. Which of the following antiarrhythmic drug is contraindicated in a patient with interstitial lung disease? A. Amiodarone B. Sotalol C. Quinidine D. Lignocaine
  • 271. 270. Which of the following is a late inward sodium channel blocker? A. Ivabradine B. Ranolazine C. Trimetazidine D. Fasudil
  • 272. 271. A patient of biliary colic presented to hospital. Intern gave an injection and the pain worsened. Which is the most likely injection given? A. Morphine B. Diclofenac C. Nefopam D. Etoricoxib
  • 273. 272. Drug of choice for post menopausal osteoporosis is A. Estrogen B. Bisphosphonates C. Teriparatide D. Thyroxine
  • 274. 273. Which of the following is not used as emergency contraceptive? A. Danazol B. Levonorgestrel C. Mifepristone D. IUCD
  • 275. 274. As an Indian medical intern, which of the following is the correct format for prescription of alprazolam? a. Tablet alprazolam 0.5 mg once a day before bed time for 7 days b. Tablet alprazolam 0.5 mg HS for 7 days c. Tablet alprazolam 500 mcg one tablet OD for 7 days d. Tablet alprazolam ½ mg tablet HS daily
  • 276. 275. Rate of administration of a drug is equal to rate of elimination. How will you calculate the dosing rate of the drug to maintain steady state concentration? a. Dosing rate = Vd x target plasma concentration b. Dosing rate = CL x target plasma concentration c. Dosing rate = Vd/target plasma concentration d. Dosing rate = CL/target plasma concentration
  • 277. 276. A patient with bronchial asthma presents with raised intraocular pressure. Treatment of open angle glaucoma in this patient is a. Latanoprost b. Alprostadil c. Gemeprost d. Carboprost
  • 278. 277. A patient presented with suspected cocaine overdose. Which of th following features is not seen in this patient? a. Bradycardia b. Agitation c. Myocardial infarction d. Hyperthermia
  • 279. 278. A child presents to hospital with agitation, photophobia and retention of urine. History reveals that he has eaten a wild fruit given by his friend. Which of the following substances is the likely cause of poisoning and its antidote respectively? a. Dhatura, Physostigmine b. Dhatura, Pralidoxime c. Yellow Oleander, Physostigmine d. Yellow Oleander, Pralidoxime
  • 280. 279. A 45-year-old patient being treated with low dose aspirin since 6 months presented with rectal bleeding. Inhibition of which of the following substance is likely to responsible for the bleeding? a. TXA2 b. LT c. Bradykinin d. PGI2
  • 281. 280. A 34-week pregnant female with polyhydramnios presents with labour pain. She was treated with indomethacin earlier. Which of the following can be likely outcome in the baby if delivery occurs at this time? a. Patent ductus arteriosus b. Premature closure of ductus arteriosus c. Patent ductus venosus d. Premature closure of ductus venosus
  • 282. 281. A 60-year-old female with renal disease was admitted for pyelolithotomy. Post operative analgesic of choice in this patient is? a. Diclofenac b. Indomethacin c. Naproxen d. Acetaminophen
  • 283. 282. A 45-year-old patient presented with the symptoms shown in image below. Patient is a known hypertensive and was taking some antihypertensive drug. Which of the following drug is the likely cause of this condition? a. Lisinopril b. Clonidine c. Nifedipine d. Timolol
  • 284. 283. Tocilizumab is an immunosuppressive drug used for Rheumatoid arthritis. It is a monoclonal antibody against a. IL-6 b. IL-12 c. IL-2 d. IL-3
  • 285. 284. A patient on digoxin therapy and had a level of 1 ng/ml. He was receiving several other drugs and presented 2 months later with flattening of T waves in ECG. Now the plasma level of digoxin is 3.8 ng/ml. Which of the following drug is likely to be responsible? a. Triamterene b. Atenolol c. Clarithromycin d. KCl
  • 286. 285. A 26-year-old female came to hospital for pre-conceptional counselling. Patient is a known hypertensive compliant to drug therapy. Which of the following drugs will you advise to stop? a. Methyl dopa b. Labetalol c. Lisinopril d. Nifedipine
  • 287. 286. A 60-year-old post-menopausal woman having a previous history of Colle’s fracture presented with backache. Which of the following statements about anti-osteoporosis drugs is false regarding this patient? a. Calcium requirement is 1200 mg per day b. Oral vitamin D3 should be given along with calcium supplements c. Bisphosphonates therapy should not be given for more than one year d. Teriparatide therapy should be followed by bisphosphonate therapy
  • 288. 287. An anesthetist injected bupivacaine in a patient for axillary nerve block. Later, the patient developed restlessness, agitation and cardiovascular collapse. Next best step in the management would be? a. Cardiac resuscitation + dopamine b. Cardiac resuscitation + dantrolene c. Cardiac resuscitation + sodium bicarbonate d. Cardiac resuscitation + 20 % intralipid
  • 289. 288. A patient was admitted for surgery and halothane is being planned to be used as an anaesthetic agent. The patient was explained about the adverse effect of malignant hyperthermia with this drug. Which of the following drugs may also result in malignant hyperthermia? a. Succinylcholine b. D-Tubocurarine c. Dantrolene d. Baclofen
  • 290. 289. A patient presented to hospital with three bouts of vomiting and treated with anti-emetic drug. Vomiting subsided but after sometime, he developed abnormal movements. Which of the following drug is used for treatment of these motor symptoms? a. Benzhexol b. Cyproheptadine c. L-dopa d. Hyoscine
  • 291. 290. A female taking oral contraceptives acquired tuberculosis. After prescribing anti-tubercular therapy, physician advised the patient for alternative contraception. What is the probable reason of this advise? a. Rifampicin causes teratogenicity b. Isoniazid is teratogenic c. Rifampicin increases the metabolism of oral contraceptives d. Oral contraceptives decrease the efficacy of anti-tubercular therapy
  • 292. 291. Metronidazole is used for treatment of various anaerobic infections as well as bacterial vaginosis. Patient is instructed to avoid taking which of the following substance after metronidazole prescription to avoid possible adverse effects? a. Grapefruit juice b. Milk c. Alcohol d. Orange juice
  • 293. 292. A patient of borderline leprosy received multi drug therapy (MDT). He developed skin lesions as shown in the figure below. How should this patient be managed? a. Stop MDT and give thalidomide b. Continue MDT and give thalidomide c. Stop MDT and give steroids d. Continue MDT and give steroids
  • 294. 293. A 20-year-old boy presented with slow growing mass on the medial side of thigh. Investigations revealed it to be osteosarcoma and patient was started on methotrexate. Mechanism of action of methotrexate as an anticancer drug is a. Inhibition of Dihydrofolate reductase b. Inhibition of N5 tetrahydro folate reductase c. Inhibition of tyrosine kinase d. Inhibition of purine metabolism
  • 295. 294. Topical application of 400 mcg/ml mitomycin C is used for a. Subglottic stenosis b. Tympanoplasty c. Rhinocerebral mucormycosis d. Nasopharyngeal bleeding